BRS Behavioral Science/Drugs

Lakukan tugas rumah & ujian kamu dengan baik sekarang menggunakan Quizwiz!

13. This man is at the lowest risk for suicide if he works as a (A) messenger (B) policeman (C) physician (D) lawyer (E) dentist

The answer is A Nonprofessionals are at a lower suicide risk than professionals. Among professionals, those at the highest risk for suicide are police officers, physicians, lawyers, and dentists

A 54-year-old woman with schizophrenia who has been taking a high-potency antipsychotic agent for the past 5 years has begun to show involuntary chewing and lip-smacking movements. 4. The side effect described in is best treated initially (A) by changing to a low-potency or atypical antipsychotic agent (B) with an antianxiety agent (C) with anantidepressantagent (D) with an anticonvulsant (E) by stopping the antipsychotic agent

The answer is A Tardive dyskinesia usually occurs after at least 6 months of starting a high-potency antipsy- chotic and is best treated by changing to a low-potency or atypical agent; stopping the antipsychotic medication will exacerbate the symptoms.

Since the age of 8, a 15-year-old girl with normal intelligence and interactive skills has shown a number of repetitive motor move- ments. She recently has begun to have outbursts in which she curses and shrieks. When asked if she can control the vocalizations and movements she says, "For a short time only; it is like holding your breath—eventually you have to let it out." Medical evaluation is unremarkable. 2. The most effective management of the unwanted vocalizations and movements is (A) an antipsychotic (B) an antidepressant (C) family therapy (D) a stimulant (E) individualpsychotherapy

The answer is A The most effective management for Tourette disorder is antipsychotic medication, such as haloperidol. There is no evidence that antidepressants or stimulants are helpful for control of motor or vocal tics. Psychotherapy can help patients with Tourette disorder deal with the social problems their disorder may cause, but is not the most effective management for the symptoms of the disorder.

A 23-year-old medical student comes to the emergency room with elevated heart rate, sweating, and shortness of breath. The student is convinced that she is having an asthma attack and that she will suffocate. The symptoms started suddenly during a car ride to school. The student has had episodes such as this on at least three previous occa- sions over the past 2 weeks and now is afraid to leave the house even to go to school. She has no history of asthma and, other than an increased pulse rate, physical findings are unremarkable. 2. Of the following, the most effective long- term management for this patient is (A) an antidepressant (B) a support group (C) a benzodiazepine (D) buspirone (E) aβ-blocker

The answer is A While the most effective immediate treatment for this patient is a benzodiazepine because it works quickly, the most effective long-term (main- tenance) management is an antidepressant, particularly a selective serotonin reuptake inhibitor (SSRI) such as paroxetine (Paxil).

A 30-year-old patient is brought to the emergency department after being found running down the street naked. He is speak- ing very quickly and tells the physician that he has just given his clothing and all of his money to a homeless man. He states that God spoke to him and told him to do this. His history reveals that he is a prac- ticing attorney who is married with three children. 17. The most effective long-term management for this patient is (A) lithium (B) fluoxetine (C) amitriptyline (D) diazepam (E) haloperidol

The answer is A hile the most effective immediate treatment for this patient is a fast-acting, high-potency antipsychotic agent, such as haloperidol, to control his hallucinations and delusions, lithium, which takes 2-3 weeks to work, would be more effective for long-term maintenance. Fluoxetine, amitriptyline, and diazepam are less appropriate primary treatments for this bipolar patient.

3. A 4-year-old child who has never spoken voluntarily shows no interest in or connection to his parents, other adults, or other children. Medical examination and otological testing are unremarkable. The child's mother tells the doctor that he persistently turns on the taps to watch the water running and that he screams and struggles fiercely when she tries to dress him. Which of the following disorders best fits this clinical picture? (A) ASD (Autism) (B) Rett disorder (C) ADHD (D) Tourette disorder (E) Selective mutism

The answer is A. This child, who has never spoken voluntarily and who shows no interest in or connection to his parents, other adults, or other children despite normal hearing, probably has ASD. He turns on the tap to watch the water running because, as with many children with ASD, repetitive motion calms him. Any change in his environment, such as being dressed, leads to intense discomfort, struggling, and screaming

21. What is the most appropriate antidepressant agent for rapid relief of the symptoms of depression in a 25-year-old woman? (A) Venlafaxine (B) Tranylcypromine (C) Trazodone (D) Doxepin (E) Amoxapine (F) Fluoxetine (G) Nortriptyline (H) Imipramine

The answer is A. *SNRIs may work more quickly than other antidepressants* and, as such, venlafaxine is a good choice for rapid relief of depressive symptoms in this young woman.

12. A patient sees an otolaryngologist with complaints of recurrent sinusitis. The surgeon decides to perform sinus surgery to debride the scarred sinus tissue. During the procedure, the surgeon elects to use an agent that has good local anesthesia as well as vasoconstrictive properties. What agent might he use? (A) Cocaine (B) Procaine (C) Tetracaine (D) Lidocaine (E) Mepivacaine

The answer is A. Cocaine is ideal for such surgery because of its topical activity; it does not require the addition of epinephrine, as it has intrinsic vasoconstrictive activity that aids in hemostasis. Similar to cocaine, both procaine and tetracaine are ester-type compounds; however, procaine is not topically active and tetracaine is used primarily for spinal anesthesia and ophthalmologic procedures. Lidocaine is an amide anesthetic preferred for in ltrative blocks and epidural anesthesia. Mepivacaine is another amide local anesthetic, although not topically active, which, like all such agents, acts by blocking sodium channels.

3. A 56-year-old man recently suffered a myocardial infarction (MI). He is on numerous medications, many of which are metabolized by the cytochrome P-450 system. He now presents to the psychiatrist with difficulty sleeping and decreased appetite and reports "no longer enjoying golf like I used to." Recognizing that depression is common in patients who have recently had an MI, the psychiatrist decides to start the patient on selective serotonin reuptake inhibitor (SSRI) therapy. Given his multiple medications, which SSRI should be avoided? (A) Fluoxetine (B) Tranylcypromine (C) Sertraline (D) Escitalopram (E) Phenylzine

The answer is A. Fluoxetine and paroxetine inhibit cytochromeP-450 and thus need to be used with caution as they can potentiate the action of other drugs metabolized by this system. Tranylcypromine and phenelzine are monoamine oxidase inhibitors (MAOIs) that, when used with selective serotonin reuptake inhibitors (SSRIs), can cause a potentially fatal "serotonin syndrome." Sertraline and escitalopram are two SSRIs that are not metabolized by the P-450 system and might be good choices for this individual.

7. A first-year surgery intern has rotated on numerous surgical services during the first year, including general surgery, cardiothoracic surgery, urology, surgical oncology, trauma surgery, and colorectal surgery. He has gotten quite used to liberally ordering morphine for pain control. However, which of the following is an absolute contraindication to opioid use? (A) Closed head injury (B) Myocardial infarction (C) Acute pulmonary edema (D) Renal colic (E) Biliary colic

The answer is A. Opioids are contraindicated incases of head trauma,as they increase cerebral vascular pressure and may cause hemorrhage and/or herniation. Morphine is used during myocardial infarction, as it decreases cardiac preload and chest pain. Morphine is also used to reduce dyspnea associated with acute pulmonary edema. Renal colic, pain due to a kidney stone passing through the ureter, or biliary colic, which is a similar pain associated with gallstones passing through biliary ducts, are both well managed with morphine.

10. The neurotransmitter metabolized to 5-HIAA (5-hydroxyindoleacetic acid) is (A) serotonin (B) norepinephrine (C) dopamine (D) g-aminobutyric acid (GABA) (E) acetylcholine(Ach) (F) glutamate

The answer is A. Serotonin is metabolized to 5-HIAA.

A 62-year-old patient tells the physician that he is having difficulty maintaining an erection when he has intercourse with his wife. 15. The physician recommends that the patient take sildenafil citrate (Viagra) prior to having intercourse. The major action of this agent in the management of erectile disorder is to (A) increase the concentration of cGMP (B) decrease the concentration of cGMP (C) increase the degradation of cGMP (D) increase the concentration of phosphodiesterase 5 (PDE5) (E) decrease the degradation of prostaglandin E

The answer is A. Sildenafil citrate (Viagra) works by increasing the concentration of cGMP, a vasodilator, in the penis which causes erection to persist.

15. What is the major mechanism of action of cocaine on neurotransmitter systems in the brain? (A) Blocks reuptake of dopamine (B) Blocks release of dopamine (C) Blocks reuptake of serotonin (D) Blocks release of serotonin (E) Blocks release of norepinephrine

The answer is A. The major mechanism of action of cocaine on neural systems is to block the reuptake of dopamine, thereby increasing its availability in the synapse. Increased availability of dopamine is involved in the "reward" system of the brain and the euphoric effects of stimulants.

22. A 40-year-old man tells his physician that he is often late for work because he has difficulty waking up on time. He attributes this problem to the fact that he gets out of bed repeatedly during the night to recheck the locks on the doors and to be sure the gas jets on the stove are turned off. His lateness is exacerbated by his need to count all of the traffic lights along the route. If he suspects that he missed a light, he becomes quite anxious and must then go back and recount them all. Physical examination and laboratory studies are unremarkable. Of the following, the most effective long-term man- agement for this patient is most likely to be (A) an antidepressant (B) an antipsychotic (C) a benzodiazepine (D) buspirone (E) aβ-blocker

The answer is A. This man's repeated checking and counting behavior indicates that he has OCD. The most effective long-term management for OCD is an antidepressant, particularly a selective serotonin reuptake inhibitor (SSRI) such as fluvoxamine (Luvox) or a heterocyclic agent such as clomipramine. Antianxiety agents such as benzodiazepines (e.g., diazepam) and buspirone, and β-blockers such as pro- pranolol are more commonly used for the management of acute or chronic anxiety. Antipsychotic agents such as haloperidol may be useful as adjuncts but do not substitute for SSRIs or clomipramine in OCD.

11. A 40-year-old businessman who has been a physician's patient for the past 5 years asks her for a medication to help him sleep on an overnight flight to Australia. Of the following, the best agent for this use is (A) zaleplon (B) flurazepam (C) clonazepam (D) buspirone (E) chlordiazepoxide (F) bupropion

The answer is A. Zaleplon, a non-benzodiazepine sleep agent, is the best choice to aid sleep on an overnight flight. Benzodiazepines have higher abuse potential than agents such as zaleplon. Buspirone has little abuse potential but does not cause sedation, and, in any case, takes weeks to work. Bupropion is an antidepressant agent and is non-sedating.

20. In a 50-year-old patient with schizophrenia, the size of the cerebral ventricles, glucose utilization in the frontal lobes, and size of limbic structures are most likely to be, respectively (A) increased, decreased, decreased (B) increased, decreased, increased (C) increased,increased,decreased (D) decreased, decreased, decreased (E) decreased,increased,decreased (F) decreased,increased,increased

The answer is A. In patients with schizophrenia, the size of cerebral ventricles, glucose utilization in the frontal lobes, and size of limbic structures are most likely to be increased, decreased, and decreased, respectively.

9. A 20-year-old woman tells the physician that sometimes she becomes frightened when her room is dark because her computer looks like a lion lurking in the corner. This is an example of (A) an illusion (B) a neologism (C) a hallucination (D) a delusion (E) an idea of reference

The answer is A. *An illusion is a misperception of a real external stimulus (e.g., a computer looking like a lion lurking in the corner in a darkened room).* A hallucination is a false sensory perception, and a delusion is a false belief not shared by others. An idea of reference is the false belief of being referred to by others, and a neologism is a new word invented by a psychotic person.

20. A 25-year-old man is brought to the hospital after being involved in an automobile accident in which he was driving and the other driver was killed. (A) Alcohol (B) Secobarbital (C) Cocaine (D) Methylphenidate (E) Caffeine (F) Diazepam (G) Heroin (H) Marijuana (I) Nicotine (J) Phencyclidine (PCP) (K) Lysergic acid diethylamide (LSD)

The answer is A. Alcohol use is commonly associated with automobile accidents.

A 26-year-old medical student is brought to the emergency department by her husband. The husband tells the doctor that his wife has shown odd behavior ever since failing an exam 2 weeks ago. In particular, she tells him that people are trying to poison her. The woman has no prior psychiatric history, and physical examination and laboratory results are unremarkable. 3. Analysis of neurotransmitter availability in the brain of this patient is most likely to reveal (A) increased dopamine (B) decreased dopamine (C) increased acetylcholine (D) decreased histamine (E) decreased serotonin

The answer is A. Analysis of neurotransmitter availability in the brain of this patient with a positive psychotic symptom (e.g., a delusion) is most likely to reveal *increased levels of dopamine or serotonin.* Acetylcholine and histamine are not so closely involved in the pathophysiology of psychotic symptoms.

9. The major neurotransmitter involved in the antidepressant action of fluoxetine (Prozac) is (A) serotonin (B) norepinephrine (C) dopamine (D) g-aminobutyric acid (GABA) (E) acetylcholine(Ach) (F) glutamate

The answer is A. Blockade of serotonin reuptake by presynaptic neurons is the primary action of the antidepressant fluoxetine.

9. A 25-year-old male patient who is slow moving and has a flat affect is put on fluoxetine (Prozac). Within 2 weeks, the patient is showing greatly increased activity level, flight of ideas, and pressured speech. In this patient, the medication has (A) precipitated a manic episode (B) had a toxic effect (C) had a delayed effect (D) increased anxiety (E) increased depression

The answer is A. In this depressed patient, the antidepressant fluoxetine has precipi- tated a manic episode (i.e., greatly increased activity level, flight of ideas, and pressured speech). This indicates that the patient has bipolar disorder rather than major depressive disorder. There is no evidence of increased depression, increased anxiety, or a delayed or toxic effect in this patient.

13. Which of the following psychotropic agents is most likely to be abused? (A) Diazepam (B) Haloperidol (C) Fluoxetine (D) Buspirone (E) Lithium

The answer is A. Of the listed agents, BZs such as diazepam are most likely to be abused. Antipsychotics such as haloperidol, antidepressants such as fluoxetine, mood stabilizers such as lithium, and non-benzodiazepines such as buspirone have little or no abuse potential.

22. Remembering that school closes early before Thanksgiving Day every year is an example of which of the following types of memory? (A) Semantic (B) Episodic (C) Procedural (D) Working

The answer is A. Remembering that school closes early before Thanksgiving Day every year is an example of semantic memory. Semantic memory is a type of declarative memory which involves remembering general knowledge about the world. Episodic memory involves remembering personally experienced events, procedural memory involves remembering things one does automatically, and working memory involves remembering recent information.

11. A 20-year-old man who has been drinking eight cups of coffee a day for the past week comes in for a physical examination. At this time, this man is most likely to show (A) tachycardia (B) decreased peristalsis (C) weight gain (D) fatigue (E) headache

The answer is A. Tachycardia, increased peristalsis, increased energy, and decreased appetite are physical effects of stimulants like caffeine. Headaches are more likely to result from withdrawal rather than use of stimulant drugs.

2. A 75-year-old man with a 3-year history of Alzheimer disease has recently become disoriented when the lights are turned off at night. He wanders about the apartment at night and his wife is concerned that he will injure himself while she is sleeping. The Folstein Mini-Mental State Exam shows that the patient is disoriented regarding time and place and has poor short-term memory. Physical examination is unremarkable and the patient is not currently taking any medication. What is the most appropriate first recommendation for the management of this patient? (A) Ask the wife to increase home nighttime lighting (B) Prescribe donepezil for the patient (C) Prescribe haloperidol for the patient (D) Prescribe methylphenidate for the wife so that she can stay alert during the night (E) Recommend that the patient be put in mechanical restraints at bedtime

The answer is A. The most appropriate first recommendation for the management of this patient is to ask the wife to increase home nighttime lighting. Lighting will improve the patient's ability to negotiate the apartment at night and so reduce his nocturnal disorientation. Keeping the wife awake is not practical or positive for her and mechanical restraints should be avoided if possible

24. Concerned parents tell the doctor that last week their 6-year-old daughter walked in on them while they were having intercourse. This week they found their daughter with another 6-year-old girl playing the male role in what looked like a simulation of sexual intercourse. Physical exam is normal. The next step for the doctor to take is to (A) Reassure the parents that this behavior is typical in children of this age (B) Tell the parents that the child is likely to have a lesbian sexual orientation in adulthood (C) Refer the family to a child psychiatrist (D) Evaluate the child's sex hormone levels (E) Advise the parents that the child probably has been sexually abused

The answer is A. The next step for the doctor to take is to reassure the parents that curiosity about sexual behavior is typical for children of this age. Although the parents should speak to the child to be sure that she has not been sexually abused by an adult, her behavior with the other girl can be explained as an attempt to replicate the behav- ior of her parents. Since sexual orientation is primarily biological, there is no reason to believe that observing sexual behavior or playing the male role in a sexual game will result in the child's having a lesbian sexual orientation. Since physical examination is unremarkable, the child's sex hormone levels are likely to be normal.

3. The mother of an obese 12-year-old boy tells the physician that the "child is not eating well." What is the physician's best response to the mother's statement? (A) What do you mean by "not eating well"? (B) The child looks like he is eating well enough. (C) There are a number of diets available that are excellent for children. (D) Increased exercising may be the answer to your son's weight problem. (E) Diet plus exercise is the most effective management for obesity.

The answer is A. The physician's best response to the mother's statement is to get more information, for example, "What do you mean by not eating well?" Recommending changes in diet or exercise or commenting on the child's appearance are not appropriate until you find out more about the mother's perception of the problem.

7. A 50-year-old man shows breast enlargement after years of abusing a substance. The substance he is most likely to have abused is (A) alcohol (B) marijuana (C) heroin (D) amphetamine (E) amylnitrite

The answer is A. The substance that this 50-year-old man with breast enlargement is most likely to have abused is alcohol. Long-term use of alcohol damages the liver, resulting in accumulation of estrogens and feminization of the body. Marijuana, heroin, amphetamine, and amyl nitrite are much less likely to cause estrogen accumulation.

20. A 20-year-old female college student tells the doctor that because she was afraid to be alone, she tried to commit suicide after a man with whom she had had two dates did not call her again. After the interview, she tells him that all of the other doctors she has seen were terrible and that he is the only doctor who has ever understood her problems. (A) Borderline personality disorder (B) Histrionic personality disorder (C) Obsessive-compulsive personality disorder (D) Avoidant personality disorder (E) Antisocial personality disorder (F) Dependent personality disorder (G) Dissociative identity disorder (H) Paranoid personality disorder (I) Passive-aggressive personality disorder (J) Narcissistic personality disorder (K) Schizotypal personality disorder (L) Schizoid personality disorder

The answer is A. This 20-year-old college student, who made a suicide attempt after a relatively trivial relationship broke up and who uses splitting as a defense mechanism (e.g., all of the other doctors she has seen were terrible and this doctor is perfect), is showing evidence of borderline personality disorder.

8. After starting first grade, a 7-year-old boy often complains of feeling ill and refuses to go to school. There are no medical findings. At home, the child is appropriately interactive with his parents and, when friends visit, he plays well with them. At first his parents let him stay at home, but they are becoming increasingly concerned that he is falling behind in his schoolwork. The parents want to hire a home tutor for the child. What is the pediatrician's next step in management? (A) Advise the parents to go to school with the child and, over days, gradually decrease the time they spend there. (B) Advise the parents to allow the child to stay at home until he indicates that he is comfortable separating from the parents. (C) Advise the parents to ignore the school refusal, bring the child to school, and tell him what time they will pick him up. (D) Reassure the parents that hiring a home tutor for the current school year is best for the child. (E) Prescribe an anti anxiety agent for the child to be given only on school days.

The answer is A. This child is showing evidence of separation anxiety disorder. By the age of 3 to 4 years children should be able to spend some time away from parents in a school setting. The pediatrician's best recommendation is for the parents to go to school with the child and, over days, gradually decrease the time they spend there. Allowing the child to stay at home or hiring a home tutor will just increase the child's difficulty separating from his parents. Pharmacologic therapy is not the first choice in the management of this child.

4. A 27-year-old patient with schizophrenia shows extreme psychomotor agitation to the point of physical exhaustion. At times, he holds unusual, uncomfortable-looking body positions. This patient is most likely to have which of the following subtypes of schizophrenia? (A) Catatonic (B) Disorganized (C) Paranoid (D) Residual (E) Undifferentiated

The answer is A. This patient who shows *extreme psychomotor agitation and unusual uncomfortable-looking body positions* is most likely to have *catatonic schizophrenia.* Disorganized schizophrenia is characterized by disinhibition, poor grooming, poor personal appearance, and inappropriate emotional responses. Paranoid schizophrenia is characterized by delusions of persecution; undifferentiated schizophrenia has the characteristics of more than one subtype. In residual schizophrenia, there is one previous psychotic episode and residual symptoms, but no current psychotic symptoms.

16. After 20 years of smoking, a 45-year-old female patient has decided to quit. Of the following, what physical effect is most likely to be seen as a result of this patient's withdrawal from nicotine? (A) Weight gain (B) Euphoria (C) Excitability (D) Delirium tremens (E) Long-term abstinence

The answer is A. Weight gain commonly occurs following withdrawal from *stimulants such as nicotine*. Mild depression of mood and lethargy are also seen. Long-term abstinence is uncommon in smokers; most smokers who quit relapse within 2 years. Delirium tremens occur with withdrawal from sedatives such as alcohol.

7. Which of the following symptoms of schizophrenia is likely to respond best to antipsychotic medication? (A) Delusions (B) Flattening of affect (C) Poor speech content (D) Lack of motivation (E) Social withdrawal

The answer is A. When compared to negative symptoms (e.g., flattening of affect, poor speech content, lack of motivation, and social withdrawal), positive symptoms such as delusions respond better to antipsychotic medication.

17. When compared with a man, the chances that a woman will develop major depressive disorder, dysthymic disorder, or bipolar disorder over the course of her lifetime are, respectively (A) higher, higher, equal (B) higher, higher, lower (C) higher,equal,higher (D) higher, higher, higher (E) equal, higher, equal (F) equal, higher, lower (G) equal, equal, equal

The answer is A. When compared with a man, a woman is twice as likely to develop major depressive disorder, and three times as likely to develop dysthymic disorder. Bipolar disorder and cyclothymic disorder occur equally in men and women.

A 25-year-old patient who has taken haloperidol for the past 2 months is brought to the hospital with a temperature of 104°F, blood pressure of 190/110, and muscular rigidity. 5. These signs indicate that the patient has an antipsychotic medication side effect known as (A) restless legs syndrome (B) neuroleptic malignant syndrome (C) akathisia (D) tardive dyskinesia (E) acute dystonia (F) pseudoparkinsonism

The answer is B High body temperature and blood pressure, and mus- cular rigidity indicate that the patient has developed an antipsychotic medication side effect known as neuroleptic malignant syndrome

A 20-year-old male college student is taken to the emergency department by police because he tried to enter a state office building to "have a conference with the governor" about conducting a fund drive to "finance my cure for cancer." When police prevent him from entering the building, he becomes irritable and hostile and resists attempts to restrain him 3. The most effective long-term management for this patient is (A) a heterocyclic antidepressant (B) lithium (C) electroconvulsive therapy (D) psychotherapy (E) a monoamine oxidase inhibitor

The answer is B Of the listed treatments, the one most effective for bipolar disorder is lithium. Heterocyclic antidepressants, electroconvulsive therapy, monoamine oxidase inhibitors, and psychotherapy are used primarily to manage depression. Antidepressants and psychotherapy are used to manage dysthymia

For the past few months, a 28-year-old woman has seemed full of energy and optimism for no obvious reason. Although she gets only about 6 hours of sleep a night, she has been very productive at work. She is talkative and gregarious and relates that she belongs to four clubs and two different sports teams. A few years previously, friends say she was often pessimistic and seemed tired and "washed out." During that period, she continued to work but did not seek out social activities and had little interest in sex. There is no evidence of a thought disorder and the patient denies suicidality or hopelessness. Physical examination including body weight is normal. 6. The most effective long-term management for this patient is (A) a heterocyclic antidepressant (B) lithium (C) electroconvulsive therapy (D) psychotherapy (E) a monoamine oxidase inhibitor

The answer is B Of the listed treatments, the one most effective for cyclothymic disor- der, as for bipolar disorder, is lithium. Heterocyclic antidepressants, electroconvulsive therapy, monoamine oxidase inhibitors, and psychotherapy are primarily used to manage depression. Antidepressants and psychotherapy are used to manage dysthymia.

A 78-year-old retired female physician reports that she has been confused and forgetful over the past 10 months. She also has difficulty sleeping, her appetite is poor, and she has lost 20 pounds. Questioning reveals that her 18-year-old dog died 10 months ago. 5. At this time, the most appropriate diagnosis for this patient is (A) delirium (B) pseudodementia (C) Alzheimer disease (D) dissociative fugue (E) amnestic disorder

The answer is B The best explanation for this patient's symptoms is pseudodementia—depression that mimics dementia. In the elderly, depression is often associated with cognitive problems as well as sleep and eating problems. Evidence for depression is provided by the fact that this patient's symptoms began with the loss of an important relationship (i.e., the death of her dog). Delirium and dementia are caused by physiological abnormalities. Dissociative fugue involves wandering away from home, and amnestic disorder is associated with a history of alcoholism.

A 20-year-old male college student is taken to the emergency department by police because he tried to enter a state office building to "have a conference with the governor" about conducting a fund drive to "finance my cure for cancer." When police prevent him from entering the building, he becomes irritable and hostile and resists attempts to restrain him. 4. This college student has two brothers. The first is his monozygotic twin; the second is 2 years younger. The risks that his first and second brothers will develop bipolar disorder are respectively about (A) 75% and 60% (B) 75% and 20% (C) 60% and 10% (D) 50% and 10% (E) 10% and 1%

The answer is B The chances of the monozygotic twin and first-degree relative (e.g., brother) of this bipolar patient developing the disorder are about 75% and 20%, respectively.

A 75-year-old man is brought to the emer- gency department after being burned in a house fire. This is the patient's third emer- gency visit in 2 months. His other visits occurred after he inhaled natural gas when he left the stove on without a flame, and because he fell down the stairs after wander- ing out of the house in the middle of the night. There is no evidence of physical illness and no history of substance abuse. His wife is distressed and begs the doctor to let her hus- band come home. 8. Of the following, the most appropriate initial intervention for this patient is (A) antipsychotic medication (B) provision of a structured environment (C) antidepressant medication (D) donepezil (E) reassurance

The answer is B The most effective initial intervention for this patient is provision of a structured environment (e.g., giving the patient visual cues for orientation [labeling doors for function]) and taking practical measures (e.g., removing the gas stove). Donepezil can then be used to slow the progression of his illness. Other medications and reassurance may be useful for symptoms such as psychosis, depression, and anxiety, but will have little effect on the patient's forgetful and potentially dangerous behavior.

A 9-year-old boy with normal intelligence frequently gets into trouble at school because he blurts out answers, interrupts the teacher, disturbs the other students, and cannot seem to sit still in class. He also frequently injures himself during play and rarely sits through an entire meal at home. His siblings say that he is "a real pest." However, the child does his schoolwork well and behaves well when he is alone with his tutor. 4. The best explanation for this child's behavior is (A) oppositional defiant disorder (B) ADHD (C) social difficulties in the family (D) conduct disorder (E) typical,age-appropriatebehavior

The answer is B This 9-year-old boy who gets into trouble at school because he disturbs the teacher and the other stu- dents, has behavioral difficulties at home and with siblings, and cannot seem to sit still is showing evidence of ADHD (see also answer to Question 1). Children with ADHD can often learn well when there are few distractions (e.g., alone with a tutor). Children with conduct disorder show behavior that violates social norms (e.g., stealing). In contrast, children with ADHD have trouble controlling their behavior but do not intentionally cause harm. Children with oppositional defiant disorder have problems dealing with authority figures but not with other children or animals.

1. A 42-year-old businessman visits a psychiatrist for what he describes as a very "embarrassing problem." The patient has found it difficult to make it to work on time because he keeps driving back to his house to make sure that the garage door is shut. He has begun waking up 2 hours early to facilitate these obsessions and compulsions. He is otherwise without additional complaints. The psychiatrist is concerned that the patient has developed obsessive-compulsive disorder (OCD) and has him try which of the following? (A) Imipramine (B) Clomipramine (C) Atomoxetine (D) Propranolol (E) Desipramine

The answer is B. Clomipramine,a tricyclic antidepressant (TCA)thatinhibitsserotonin uptake as well as the selective serotonin-reuptake inhibitors (SSRIs), is used for the treatment of obsessive-compulsive disorder (OCD). Imipramine is a tricyclic agent that is used infrequently to suppress enuresis in children and that is metabolized to yet another TCA, desip- ramine. Atomoxetine is a selective inhibitor of the norepinephrine reuptake transporter that is used in the management of attention-deficit disorders. Propranolol is a β-blocker used to treat hypertension and other cardiovascular conditions.

23. A 55-year-old patient who is taking tiotropium bromide (Spiriva) for chronic obstructive pulmonary disease has memory problems due to the agent's action on which of the following receptors? (A) Adrenergic (B) Cholinergic (C) Dopaminergic (D) Histaminergic (E) Serotonergic

The answer is B. Decreased availability of acetylcholine by blockade of muscarinic acetylcholine receptors (i.e., anticholinergic activity) in the CNS is associated with memory problems. Blockade of adrenergic, dopaminergic, histaminergic, and serotonergic receptors are not specifically associated with memory problems.

10. A 16-year-old patient visits his dentist for a routine checkup. He finds that his wisdom teeth are severely impacted and need to be removed. The oral surgeon to whom he is referred plans on using an agent that has good analgesic and sedative properties but does not cause skeletal muscle relaxation. Which agent has these ideal properties? (A) Enflurane (B) Nitrous oxide (C) Thiopental (D) Halothane (E) Isoflurane

The answer is B. Nitrous oxide is an anesthetic gas that has good analgesic and sedative properties without the skeletal muscle-relaxing effects. It can be used along with other inhaled agents decreasing their concentrations and thus their side effects. Enflurane produces anesthesia, hypnosis, and muscle relaxation, as do the others, and is very pungent. Thiopental is a barbiturate that is too short acting for this application. Halothane has a pleasant odor and produces a smooth and relatively rapid induction but decreases cardiac output and can result in an unpredictable hepatotoxicity. Isoflurane is associated with more rapid induction and recovery than halothane and may have some benefits in patients with ischemic heart disease.

A 9-year-old boy with normal intelligence frequently gets into trouble at school because he blurts out answers, interrupts the teacher, disturbs the other students, and cannot seem to sit still in class. He also fre- quently injures himself during play and rarely sits through an entire meal at home. His siblings say that he is "a real pest." However, the child does his schoolwork well and behaves well when he is alone with his tutor. 6. Of the following, the most effective management for this child is (A) lithium (B) a stimulant (C) an antidepressant (D) a sedative (E) psychotherapy

The answer is B. The most effective management for children with ADHD is use of central nervous system stimulants including methylphenidate (Ritalin), and dex- troamphetamine sulfate (Dexedrine). Lithium is used to treat bipolar disorder, antide- pressants are used primarily to treat depression, and sedatives are used primarily to treat anxiety. While psychotherapy may help the parents and child deal with the behavioral symptoms, it is not the most effective management since the disorder is caused by neurologic dysfunction

24. A 6-year-old child shows seizures, cognitive defects, and autistic behavior. The child also shows raised discolored areas on her forehead (forehead plaques). Which of the following chromosomes is most likely to be involved in the etiology of this child's symptoms? (A) 1 (B) 16 (C) 18 (D) 21 (E) X

The answer is B. Chromosome 16 and chromosome 9 are both associated with tuberous sclerosis. Seizures, cognitive defects, autistic behavior, and forehead plaques in this 6-year-old child are signs of this disorder.

8. About 1 week after her final examination for a biochemistry course, a medical student's knowledge of the details of the Krebs cycle is most likely to reside in her (A) unconscious mind (B) preconscious mind (C) conscious mind (D) superego (E) ego

The answer is B. Memory of the details of the Krebs cycle, while no longer in the fore- front of the medical student's mind, can be recalled relatively easily 1 week after the examination. This memory therefore resides in the preconscious mind. The uncon- scious mind contains repressed thoughts and feelings, which are not available to the conscious mind. The conscious mind contains thoughts that a person is currently aware of. The id contains instinctive sexual and aggressive drives and is not influenced by external reality. The ego also controls the expression of the id, sustains satisfying interpersonal relationships, and, through reality testing, maintains a sense of reality about the body and the external world

11. A husband and wife in their mid-30s state that they are having sexual problems. During the interview the doctor discovers that while their sex life had been good, the last time they tried to have intercourse (4 weeks previously), the husband could not maintain an erection. Which of the following agents is most likely to have caused this |sexual problem? (A) Cocaine (B) Propranolol (C) Levodopa(L-dopa) (D) Amyl nitrite (E) Dextroamphetamine

The answer is B. Of the listed agents, the one most likely to have caused erectile disorder is propranolol, an antihypertensive medication (b-blocker). Cocaine, amphetamines, and L-dopa tend to increase sexual interest and performance by elevating dopamine availability. Amyl nitrite (a vasodilator) is used to enhance the sensation of orgasm.

6. Of the following defense mechanisms, which is considered the most mature? (A) Denial (B) Sublimation (C) Dissociation (D) Regression (E) Intellectualization

The answer is B. Sublimation, expressing an unacceptable emotion in a socially accept- able way, is classified as a mature defense mechanism. Denial, dissociation, regression, and intellectualization are all classified as less mature defense mechanisms.

4. Which of the following structures of the mind are at least partly developed in a typical 4-year-old child? (A) The id only (B) The id and the ego only (C) The id,ego,and superego (D) The ego and superego only (E) Not the id,ego,or superego

The answer is B. The id is present at birth, the ego begins to develop immediately after birth, and the superego begins to develop at about age 6 years.

14. An 80-year-old man is brought to the emergency room by his wife. The man, who has a history of depression and suicidal behavior, refuses to eat and states that life is not worth living anymore. Consultations with his primary care physician and a consulting psychiatrist reveal that the patient has not responded to at least three different antidepressant medications that he has taken in adequate doses and for adequate time periods in the past 2 years. The most appropriate next step in the management of this patient is to recommend (A) diazepam (B) electroconvulsive therapy (ECT) (C) psychotherapy (D) buspirone (E) lithium

The answer is B. The most appropriate next step is to recommend a course of electro- convulsive therapy (ECT) for this elderly, severely depressed patient. ECT is a safe, fast, effective treatment for major depression. Diazepam, lithium, buspirone, and psychotherapy will not be effective as ECT in relieving this patient's suicidal depression quickly.

29. A 32-year-old woman and her husband tell the physician that they want to have a baby but they are concerned about the impact this will have on the wife's emotional state. She has had three major depressive episodes within the past 10 years which have been treated successfully with antidepressant medication. The woman is concerned that she may become depressed while pregnant and will be unable to take any medication because of the pregnancy. Which of the following is the most appropriate response by the physician? (A) "Depression associated with pregnancy is unrelated to major depression; therefore, you are not at greater risk for being depressed while pregnant." (B) "The risk for depression is greatest after delivery and depression during pregnancy can often be safely treated." (C) "The risk for depression is greatest during pregnancy, but ECT is quite safe." (D) "Since you have been symptom-free for the past year, you should not be at great- est risk for depression than the normal population." (E) "We will need to follow you closely since the suicide rate is higher for women who are pregnant than for women who are not pregnant."

The answer is B. The most appropriate response for the physician is to tell the patient that the risk for depression is greater after than before delivery and that depression during pregnancy can often be safely treated. Most antidepressants are in pregnancy Category C but two, bupropion and maprotiline, are in Category B. Discussing differential suicide rates is not a helpful intervention. In any case the suicide rate is lower for women who are pregnant than for those who are not pregnant. While ECT is quite safe in pregnancy, psychopharmacology is less invasive and usually preferred.

9. A 60-year-old man is brought to the hospital after a fall outside of a neighborhood bar. Radiologic studies indicate that the patient has a fractured hip and surgery is performed immediately. Two days later, the patient begins to show an intense hand tremor and tachycardia. He tells the doctor that he has been "shaky" ever since his admission and that the shakiness is getting worse. The patient states that while he feels frightened, he is comforted by the fact that the nurse is an old friend (he has never met the nurse before). He also reports that he has started to see spiders crawling on the walls and can feel them crawling on his arms. The doctor notes that the patient's speech seems to be drifting from one subject to another. Of the following, what is the most likely cause of this picture? (A) Alcohol use (B) Alcohol withdrawal (C) Heroin use (D) Heroin withdrawal (E) Amphetamine withdrawal

The answer is B. The most likely cause of tremor, tachycardia, illusions (e.g., believing the nurse is an old friend), and visual and tactile hallucinations (e.g., formication—the feeling of insects crawling on the skin) in this patient is alcohol withdrawal, since the use of alcohol during the past few days of hospitalization is unlikely (see also the TBQ). His fractured hip may have been sustained in the fall while he was intoxicated. Heroin use and heroin and amphetamine withdrawal generally are not associated with psychotic symptoms.

14. Two days after a 23-year-old man is rescued from a burning building he has no memory of the fire or of the few hours before or after it. Physical examination is unremarkable. The most likely explanation for this clinical picture is (A) posttraumatic stress disorder (B) dissociative amnesia (C) adjustment disorder (D) early onset Alzheimer disease (E) subarachnoid hemorrhage

The answer is B. The most likely explanation for this clinical picture, for example, having no memory of a traumatic event with no physical findings, is dissociative amnesia. In PTSD and in adjustment disorder there is no frank memory loss. Subarachnoid hemor- rhage, a hemorrhage in the space between the arachnoid space and the pia mater, typi- cally presents with a "thunderclap" headache, vomiting, or other neurologic symptoms.

25. A 72-year-old man with Alzheimer disease is being treated with memantine. What is believed to be the basis of the therapeutic action of memantine on neurons in the brain? (A) To inhibit the action of acetylcholinesterase (B) To block the influx of calcium (C) To inhibit the action of acetylcholine (D) To increase the influx of glutamate (E) To facilitate the influx of calcium

The answer is B. The therapeutic action of memantine is believed to be to decrease the influx of glutamate which ultimately blocks the influx of calcium which can lead to nerve cell degeneration and death. In contrast to a group of drugs also used to treat Alzheimer's, that is, the acetylcholinesterase inhibitors, memantine does not directly affect acetylcholine.

21. Whenever a 28-year-old woman comes to the doctor's office, she brings gifts for the receptionist and the nurses. When she hears that one of the nurses has taken another job, she begins to sob loudly. When the doctor sees her, she reports that she is so warm that she must have "a fever of at least 106°F." (A) Borderline personality disorder (B) Histrionic personality disorder (C) Obsessive-compulsive personality disorder (D) Avoidant personality disorder (E) Antisocial personality disorder (F) Dependent personality disorder (G) Dissociative identity disorder (H) Paranoid personality disorder (I) Passive-aggressive personality disorder (J) Narcissistic personality disorder (K) Schizotypal personality disorder (L) Schizoid personality disorder

The answer is B. This 28-year-old woman who brings gifts for the receptionist and the nurses because she needs to have everyone pay attention to her is showing evidence of histrionic personality disorder. Patients with this personality disorder tend to exaggerate their physical symptoms for dramatic effect (e.g., "a fever of at least 106°F").

12. A 65-year-old married couple complains to the doctor that their sex life is not what it used to be. Which of the following problems is the couple most likely to report? (A) Premature ejaculation (B) Vaginal dryness (C) Shorter refractory period (D) Decreased sexual interest (E) Vaginismus

The answer is B. This 65-year-old married couple is most likely to be having sexual problems because of vaginal dryness due to lack of estrogen after menopause. Aging is also characterized by a longer refractory period and delayed ejaculation in men and decreased intensity of orgasm in men and women. Although sexual behavior may decrease with aging because of these problems, sexual interest remains about the same. Vaginismus is not particularly associated with aging.

14. An 80-year-old female patient has a resting tremor of her left hand, little expression in her face, and problems taking a first step when she has been standing still. The area(s) of the brain most likely to be affected in this patient is (are) the (A) right parietal lobe (B) basal ganglia (C) hippocampus (D) reticular system (E) amygdala (F) left frontal lobe

The answer is B. This 80-year-old female patient is showing signs of Parkinson disease (e.g., a resting tremor, little facial expression, and problems initiating movement). This disorder is associated with abnormalities of the basal ganglia.

14. A 41-year-old man says that he has been "sickly" for most of his life. He has seen many doctors but is angry with most of them because they ultimately referred him for psychological help. He now fears that he has stomach cancer because his stomach makes noises after he eats. Physical examination is unremarkable and body weight is normal. (A) Post-traumatic stress disorder (B) Hypochondriasis (C) Obsessive-compulsive disorder (D) Panic disorder (E) Somatization disorder (F) Generalized anxiety disorder (G) Body dysmorphic disorder (H) Conversion disorder (I) Specific phobia (J) Social phobia (K) Adjustment disorder (L) Masked depression

The answer is B. This man, who says that he has been "sickly" for most of his life and fears that he has stomach cancer, is showing evidence of hypochondriasis, exaggerated concern over normal physical sensations (e.g., stomach noises) and minor ailments. There are no physical findings nor obvious evidence of depression in this patient.

A 36-year-old patient with schizophrenia tells the physician that the government has been listening in on all of his phone conversations for the past year. 6. The patient's false belief about the government is an example of a disorder of (A) thought processes (B) thought content (C) form of thought (D) perception (E) affect

The answer is B. This paranoid belief is a delusion, an example of a disorder of thought content. An idea of reference is also an example of a disorder of thought content. Illusions and hallucinations are disorders of perception, and loose associations and tangentiality are disorders of form of thought. Problems with affect are more characteristic of the mood disorders but also are seen in schizoaffective disorder.

10. A 35-year-old physician tells his internist that he has lost interest in playing in the hospital string quartet, an activity he formerly enjoyed. He reports that over the past 3 months he commonly wakes up a few hours before his alarm goes off and cannot fall back to sleep, and has lost 12 pounds without dieting. He states "maybe my family would be better off without me." He says that although he has lots of aches and pains and often feels tired, he feels somewhat better as the day progresses. Physical examination and laboratory studies are unremarkable. The most appropriate diagnosis for this patient is (A) dysthymic disorder (B) major depressive disorder (C) masked depression (D) hypochondriasis (E) cyclothymicdisorder (F) malingering (G) bipolar disorder

The answer is B. This patient is most likely to have major depressive disorder. Evidence for this is that, while there are no physical findings, he has lost interest in his usual activities, wakes up too early in the morning, has vague physical symptoms, shows diurnal variation in symptoms (worse in the morning), has lost a significant amount of weight, and is showing suicidal ideation (e.g., "maybe my family would be better off without me"). Also, his symptoms have been present for a discrete, identified amount of time. Dysthymic disorder involves mild or moderate depression most of the time, occurring over a 2-year period with no discrete episodes of illness. Bipolar disorder involves episodes of both mania and depression. Cyclothymic disorder involves episodes of hypomania and dysthymia occurring over a 2-year period with no discrete episodes of illness. In hypochondriasis, patients believe that normal body functions or minor illnesses are serious or life threatening. People who are malingering fabricate symptoms for obvious gain (e.g., to win a lawsuit) (see Chapter 13).

4. A 35-year-old woman who was raped 5 years ago has recurrent vivid memories of the incident accompanied by intense anxiety. These memories frequently intrude during her daily activities, and nightmares about the event often wake her. Her symptoms intensified when a coworker was raped 2 months ago. Of the following, the most effective long-term management for this patient is (A) an antidepressant (B) a support group (C) a benzodiazepine (D) buspirone (E) aβ-blocker

The answer is B. This patient is most likely to have post-traumatic stress disorder (PTSD). This disorder, which is characterized by symptoms of anxiety and intrusive memories and nightmares of a life-threatening event such as rape, can last for many years in chronic form and may have been intensified in this patient by re-experiencing her own rape through the rape of her coworker. The most effective long-term management for this patient is a support group, in this case a rape survivor's group. Pharmacologic treatment is useful as an adjunct to psychological management in PTSD.

24. A 35-year-old man who has been taking haloperidol for the last year develops a resting tremor, mask-like facial expression, and difficulty initiating body movements. After reducing the haloperidol dose, the next step the physician should take to relieve these symptoms is to give the patient (A) a high-potency antipsychotic agent (B) an anticholinergic agent (C) an anti anxiety agent (D) an antidepressant agent (E) lithium

The answer is B. This patient is showing evidence of pseudoparkinsonism, a neurologic side effect caused by excessive blockade of postsynaptic dopamine receptors during treatment with high-potency antipsychotics, such as haloperidol. Because dopamine normally suppresses acetylcholine activity, giving the patient an anticholinergic agent (e.g., benztropine) will serve to increase dopaminergic activity and relieve the patient's symptoms. Antianxiety agents such as benzodiazepines can be used as adjuncts to anticholinergics, but antidepressants and lithium are not effective for reversing parkinsonian symptoms caused by antipsychotics.

17. A 45-year-old man with a 20-year history of severe depression and psychotic symp- toms has held different jobs, but none of them for more than 6 months. He is successfully treated for his severe depressive symp- toms, but he remains withdrawn and odd and expresses the belief that he has been "chosen" for a special mission on earth. What is the most appropriate diagnosis for this patient? (A) Schizophrenia (B) Schizoaffective disorder (C) Schizophreniform disorder (D) Brief psychotic disorder (E) Delusional disorder (F) Shared psychoticdisorder (G) Psychosis due to a general medical condition

The answer is B. This patient is showing evidence of schizoaffective disorder. This disorder is characterized by symptoms of a *mood disorder, as well as psychotic symptoms* (the delusion that he has been "chosen") as well as and lifelong social and occupational impairment (see also answer to Question 1). *Schizoaffective disorder is distinguished from a mood disorder in that the psychotic symptoms persist in the absence of mood symptoms.*

28. A 23-year-old man with a diagnosis of schizophrenia, paranoid type, has been main- tained on a low dose of haloperidol for the past 3 years. For the past month, his symptoms of paranoia and auditory hallucinations have been more prominent, and his psychiatrist decides to increase his dose of haloperidol. The patient's mother calls the psychiatrist, concerned that he seems slower than usual and has a fine resting tremor of his upper extremities. Furthermore, the patient complains that he feels "stiff." This clinical picture suggests that the patient is experiencing which of the following? (A) Benign essential tremor (B) Antipsychotic-induced parkinsonism (C) Neuroleptic malignant syndrome (D) Parkinson disease (E) Tardive dyskinesia

The answer is B. This patient who is slowed down and has a fine resting tremor of his upper extremities and stiffness is showing evidence of antipsychotic-induced parkin- sonism, often a side effect of high doses of high-potency antipsychotics such as haloperidol. Benign essential tremor and Parkinson disease are not related to antipsy- chotic medication. Although they can both be side effects of haloperidol treatment, neuroleptic malignant syndrome and tardive dyskinesia are characterized by high fever and abnormal tongue and facial movements respectively.

13. A 24-year-old patient is experiencing intense hunger as well as tiredness and headache. This patient is most likely to be withdrawing from which of the following substances? (A) Alcohol (B) Amphetamines (C) Benzodiazepines (D) Phencyclidine (PCP) (E) Heroin

The answer is B. Tiredness and headache are seen with withdrawal from stimulants. While increased appetite can be seen in withdrawal from all stimulants, the most intense hunger is seen with withdrawal from amphetamines.

12. A 40-year-old female patient who has been taking a benzodiazepine daily in moderate doses over the past 5 years abruptly stops taking the drug. When a physician sees her 2 days after her last dose, she is most likely to show (A) hypersomnia (B) tremor (C) lethargy (D) respiratory depression (E) sedation

The answer is B. Withdrawal from benzodiazepines is associated with tremor, insomnia, and anxiety. Respiratory depression and sedation are associated with the use of, not withdrawal from, sedative drugs.

19. A very anxious 25-year-old patient is examined in the emergency room. There is no evidence of physical illness. If it could be measured, the g-aminobutyric acid (GABA) activity in the brain of this patient would most likely be (A) increased (B) decreased (C) unchanged (D) higher than the activity of serotonin (E) higher than the activity of norepinephrine

The answer is B. g-Aminobutyric acid (GABA) is an inhibitory amino acid neurotransmitter in the CNS. Thus, the activity of GABA in the brain of this anxious patient is likely to be decreased. Decreased serotonin and increased norepinephrine are also involved in anxiety

12. The parents and teacher of a 7-year-old boy note that he frequently shrugs his shoulders. Often he blinks his eyes excessively and, at other times, shouts out words for no reason. In adulthood this child is at risk to develop which of the following conditions? (A) A seizure disorder (B) Obsessive-compulsive disorder (C) Conduct disorder (D) Schizophrenia (E) ASD

The answer is B. This child who shows evidence of Tourette disorder is at risk to develop obsessive-compulsive disorder (OCD) in adulthood. Both disorders involve dysfunction of the caudate nucleus. Seizure disorders, conduct disorder, schizophrenia, and ASD are not particularly associated with Tourette disorder

A 9-year-old boy with normal intelligence frequently gets into trouble at school because he blurts out answers, interrupts the teacher, disturbs the other students, and cannot seem to sit still in class. He also frequently injures himself during play and rarely sits through an entire meal at home. His siblings say that he is "a real pest." However, the child does his schoolwork well and behaves well when he is alone with his tutor. 5. Which of the following is most closely involved in the etiology of this child's problem? (A) Food allergy (B) Improper diet (C) Neurologic dysfunction (D) Excessive punishment (E) Excessive leniency

The answer is C ADHD is believed to result from neurologic dysfunction. Although anecdotal evidence has been put forward, scientific studies have not revealed an association between ADHD and either improper diet (e.g., excessive sugar intake) or food allergy (e.g., to artificial colors or flavors). The disor- der also is not a result of parenting style (e.g., excessive punishment or leniency). How- ever, in part because of their difficult behavior, children with ADHD are more likely to be physically abused by parents.

A 21-year-old ballet dancer, who is 5 feet 7 inches tall and has weighted 95 pounds (BMI = 14.5) for the past year, tells the doctor that she needs to lose another 15 pounds to pursue a career in dance. Her mood appears good. Findings on physical examination are normal except for excessive growth of downy body hair. She reports that she has not menstruated in more than 3 years. 17. Which of the following disorders is this patient at the highest risk for in the future? (A) Dermatitis (B) Osteoarthritis (C) Osteoporosis (D) Pancreatic cancer (E) Biliaryatresia

The answer is C Patients who have anorexia nervosa for an extended period (5 years in this young woman) are at high risk for osteoporosis.

A 78-year-old retired female physician reports that she has been confused and forgetful over the past 10 months. She also has difficulty sleeping, her appetite is poor, and she has lost 20 pounds. Questioning reveals that her 18-year-old dog died 10 months ago. 6. Of the following, the most appropriate initial intervention for this patient is (A) antipsychotic medication (B) provision of a structured environment (C) antidepressant medication (D) donepezil (E) reassurance

The answer is C The most effective intervention for this depressed patient is antidepressant medication. When the medication relieves the depressive symptoms, her memory will improve. Antipsychotic medication, provision of a structured environment, acetylcholinesterase inhibitors such as donepezil, and simple reassurance are not appropriate for this patient.

1. A 22-year-old man with schizophrenia who has been taking an antipsychotic for the past 3 months reports that recently he has experienced an uncomfortable sensation in his arms and legs during the day and must constantly move them. Because of this, he can sit still for only a few minutes at a time. This medication side effect is best described as (A) restless legs syndrome (B) neuroleptic malignant syndrome (C) akathisia (D) tardive dyskinesia (E) acute dystonia (F) pseudoparkinsonism

The answer is C The symptom that this patient describes is akathisia, a subjective, uncomfortable feeling of motor restlessness related to use of some antipsychotics. Restless legs syndrome also involves uncomfortable sensations in the legs, but it is a sleep disorder , which causes difficulty falling and staying asleep. Other antipsychotic side effects include neuroleptic malignant syndrome (high fever, sweating, increased pulse and blood pressure, and muscular rigidity), pseudoparkinson- ism (muscle rigidity, shuffling gait, resting tremor, and mask-like facial expression), and tardive dyskinesia (involuntary movements including chewing and lip-smacking).

A 75-year-old man is brought to the emergency department after being burned in a house fire. This is the patient's third emergency visit in 2 months. His other visits occurred after he inhaled natural gas when he left the stove on without a flame, and because he fell down the stairs after wandering out of the house in the middle of the night. There is no evidence of physical illness and no history of substance abuse. His wife is distressed and begs the doctor to let her husband come home. 7. This patient is showing evidence of (A) delirium (B) pseudodementia (C) Alzheimer disease (D) dissociative fugue (E) amnestic disorder

The answer is C This patient is showing evidence of Alzheimer disease. He is having accidents because he is forgetful (e.g., forgetting to turn off the gas jet), and wanders out of the house because he does not know which is the closet or bathroom door and which is the outside door. There is no evidence of a medical cause for his symptoms, as there would be in delirium. There is no evidence of depression, as in pseudodementia, or of a history of alcohol abuse, as in amnestic disorder.

A 23-year-old medical student comes to the emergency room with elevated heart rate, sweating, and shortness of breath. The student is convinced that she is having an asthma attack and that she will suffocate. The symptoms started suddenly during a car ride to school. The student has had episodes such as this on at least three previous occasions over the past 2 weeks and now is afraid to leave the house even to go to school. She has no history of asthma and, other than an increased pulse rate, physical findings are unremarkable. 1. Of the following, the most effective immediate treatment for this patient is (A) an antidepressant (B) a support group (C) a benzodiazepine (D) buspirone (E) aβ-blocker

The answer is C This patient is showing evidence of panic disorder with agoraphobia. Panic disorder is characterized by panic attacks, which include increased heart rate, dizziness, sweating, shortness of breath, and fainting, and the conviction that one is about to die. Attacks commonly occur twice weekly, last about 30 minutes, and are most common in young women, such as this patient. This young woman has also developed a fear of leaving the house (agoraphobia) which occurs in some patients with panic disorder.

A 39-year-old woman takes her 6-year-old son to a physician's office. She says that the child often experiences episodes of breathing problems and abdominal pain. The child's medical record shows many office visits and four abdominal surgical procedures, although no abnormalities were ever found. Physical examination and laboratory studies are unremarkable. When the doctor confronts the mother with the suspicion that she is fabricating the illness in the child, the mother angrily grabs the child and leaves the office immediately. 9. This clinical presentation is an example of (A) factitious disorder (B) conversion disorder (C) factitious disorder by proxy (D) somatization disorder (E) somatoform pain disorder (F) malingering

The answer is C This presentation is an example of factitious disorder by proxy. The mother has feigned the child's illness (episodes of breathing problems and abdominal pain) for attention from medical personnel. This faking has resulted in four abdominal surgical procedures in which no abnormalities were found. Since she knows she is lying, the mother will become angry and flee when confronted with the truth.

A 62-year-old patient tells the physician that he is having difficulty maintaining an erection when he has intercourse with his wife. 14. Which of the following illnesses is most likely to be associated with this man's problem? (A) Alzheimer disease (B) Untreated hypertension (C) Untreated diabetes (D) Myocardial infarction (E) Untreated schizophrenia

The answer is C Untreated diabetes is most likely to be associated with erectile dysfunction. Although the medications used to manage these conditions are associated with erectile dysfunction, untreated cardiac problems, hypertension, and schizophrenia are not associated with erectile dysfunction. Alzheimer disease is not associated with erectile dysfunction. In fact, sexual expression may be the last form of communication between a couple in which one partner has Alzheimer disease.

14. A 16-year-old boy is brought to the emergency room at 4 am by his friends, who report that the patient was at an all-night rave party and was agitated, hyperactive, and hypersexual. The physician learns that the boy took several pills, which his friends thought were "ecstasy" (methylenedioxymethamphetamine). Which of the following describes the mechanism of this "party" drug? (A) Antagonistic activity at the N-methyl- d-aspartate (NMDA) receptor for lutamic acid (B) Binding to the cannabinol CB1 receptor (C) Increased release of dopamine and norepinephrine (D) Mimics the action of acetylcholine (E) Agonist at post junctional serotonin receptors

The answer is C. Like other amphetamines, ecstasy increases the release of dopamine and norepinephrine. Its use is often associated with "rave" parties. Phencyclidine (PCP) is an antagonist at the N-methyl-d-aspartate (NMDA) receptor for glutamic acid, causing euphoria and hallucinations. Marijuana causes euphoria, uncontrollable laughter, loss of time perception, and increased introspection. It binds to the cannabinol CB-1 receptor. Nicotine is a powerful stimulant in tobacco products and works by mimicking the effects of acetylcholine at cholinergic nicotinic receptors. Lysergic acid diethylamide (LSD) is an agonist of the postjunctional serotonin receptors.

6. A 23-year-old man is brought to the emergency room after he was found walking the streets naked while proclaiming himself "the son of God." His urine toxicology screen comes back negative for illicit drugs or alcohol. During the interview with the on-call psychiatrist, the patient displays flight of ideas as he jumps from topic to topic. The physician recommends starting the patient on lithium therapy for acute mania. Which of the following is associated with lithium use? (A) Urinary retention (B) Weight loss (C) Fine tremor (D) A wide therapeutic margin (E) Hyperthyroidism

The answer is C. Lithium use is associated with a fine tremor that can often be successfully managed with β-blockers. Lithium is associated with polydipsia and polyuria, not urinary retention. Likewise, the other choices are opposite what might be expected with lithium use, as patients experience weight gain and hypothyroidism. Lithium has a narrow therapeutic margin, with therapeutic doses not too much lower than toxic doses.

9. A 45-year-old woman presents with the symptoms of a major depressive episode. The patient has never previously taken an antidepressant. Her physician decides to prescribe fluoxetine (Prozac). The most likely reason for this choice is that, when compared to a heterocyclic antidepressant, fluoxetine (A) is more effective (B) works faster (C) has fewer side effects (D) is less likely to be abused (E) is longlasting

The answer is C. The doctor decides to give this patient fluoxetine because, when com- pared to a heterocyclic antidepressant, SSRIs such as fluoxetine have fewer side effects. Heterocyclics and SSRIs have equal efficacy, equivalent speed of action, and equivalent length of action. Neither SSRIs nor heterocyclics are likely to be abused.

14. If this patient tries to commit suicide, the method most likely to fail is (A) shooting himself with a gun (B) crashing his car (C) slashing his wrists (D) jumping from a high place (E) hanging himself

The answer is C. The method of suicide most likely to fail is slashing the wrists or taking pills. Shooting, crashing a car, jumping from a high place, and hanging are more lethal methods of committing suicide.

A 34-year-old man has been taking fluoxetine for treatment of depression for the past 4 months. His mood is now normal but he reports that he is having sexual problems. 9. The neurotransmitter alteration most likely to be associated with this man's sexual problem is (A) increased dopamine (B) decreased dopamine (C) increased serotonin (D) decreased serotonin (E) decreased norepinephrine

The answer is C. The neurotransmitter alteration most likely to be associ- ated with delayed or absent orgasm is increased serotonin resulting from treatment with fluoxetine. Increased dopamine tends to increase sexual interest and performance. Decreased dopamine, decreased serotonin, and decreased norepinephrine are less likely to be associated with delayed orgasm than is increased serotonin.

A 20-year-old male college student is taken to the emergency department by police because he tried to enter a state office building to "have a conference with the governor" about conducting a fund drive to "finance my cure for cancer." When police prevent him from entering the building, he becomes irritable and hostile and resists attempts to restrain him. 2. The most appropriate diagnosis for this patient is (A) dysthymic disorder (B) major depressive disorder (C) bipolar disorder (D) hypochondriasis (E) cyclothymic disorder

The answer is C. This patient is most likely to have bipolar I disorder. While this disorder involves episodes of both mania and depression, a single episode of mania defines the illness. The beliefs that one is important enough to demand a conference with the governor and cure cancer are grandiose delusions. Schizophrenic delusions are commonly paranoid in nature. Irritability and hostility are also common in a manic episode.

A 45-year-old woman says that she frequently feels "nervous" and often has an "upset stomach," which includes heartburn, indigestion, and diarrhea. She has had this problem since she was 25 years of age and notes that other family members also are "tense and nervous." 5. Which of the following additional signs or symptoms is this patient most likely to show? (A) Flight of ideas (B) Hallucinations (C) Tingling in the extremities (D) Ideas of reference (E) Neologisms

The answer is C. This patient is most likely to have generalized anxiety disorder (GAD). This disorder, which includes chronic anxiety and, often, gastrointestinal symptoms is more common in women and often starts in the 20s. Genetic factors are seen in the observation that other family members have similar problems with anxiety. Additional signs or symptoms of anxiety that this patient is likely to show include tingling in the extremities and numbness around the mouth, often resulting from hyperventilation. Flight of ideas, hallucinations, ideas of reference, and neologisms are psychotic symptoms, which are not seen in the anxiety disorders or the somatoform disorders.

22. Which of the following antidepressant agents is most likely to cause extreme sedation? (A) Venlafaxine (B) Tranylcypromine (C) Trazodone (D) Doxepin (E) Amoxapine (F) Fluoxetine (G) Nortriptyline (H) Imipramine

The answer is C. Trazadone not only causes priapism but also is highly sedating. It is thus often used in patients who have depression with insomnia.

5. A 20-year-old female patient tells the doctor that she has little interest in going back to school or in getting a job. She also reports that she often craves snack food and has gained over 10 pounds in the past 4 months. What substance is this patient most likely to be using? (A) Phencyclidine (PCP) (B) Lysergic acid diethylamide (LSD) (C) Marijuana (D) Cocaine (E) Heroin

The answer is C. *The amotivational syndrome (e.g., lack of interest in getting a job or going to school) and increased appetite, particularly for snack foods, are characteristically seen in chronic users of marijuana. * Use of cocaine, heroin, phencyclidine (PCP), or lysergic acid diethylamide (LSD) may cause work-related problems, but are less likely to *increase appetite.*

16. The most common type of hallucination seen in schizophrenia is (A) visual (B) gustatory (C) auditory (D) olfactory (E) hypnagogic

The answer is C. Auditory hallucinations are the most common type of hallucinations seen in schizophrenia.

26. A 45-year-old woman with schizophrenia has been taking an atypical antipsychotic for the past year. Since starting the medication she has gained 35 pounds, has developed diabetes mellitus, and shows a prolonged QT interval. Because of these medication side effects her physician would like to switch her to a different atypical agent. Of the following atypical agents, which is likely to be the best choice for this patient? (A) Quetiapine (B) Ziprasidone (C) Aripiprazole (D) Clozapine (E) Olanzapine

The answer is C. Because of her weight gain, type 2 diabetes and cardiovascular problem, the best choice of atypical antipsychotic agent for this patient now is aripiprazole. Clozapine and olanzapine carry high risk and *ziprasidone and aripiprazole carry low risk for weight gain and diabetes.* However, ziprasidone prolongs the QT interval and so should be avoided in this patient.

20. A 24-year-old man sustains a head injury in an automobile accident. His father relates that prior to the accident, the patient was respectful, modest, controlled, and hard working. In the hospital, the patient is rude to the nurses and aides, loses his temper with the slightest provocation, and refuses to wear a hospital gown or anything else. These behavioral changes after the accident indicate that the area of the brain most likely to have been injured in this patient is the (A) dorsolateral convexity of the frontal lobe (B) hypothalamus (C) orbitofrontal cortex (D) reticular system (E) amygdala (F) nucleus basalts of Meynert

The answer is C. Behavioral changes such as decreased impulse control, poor social behavior, and lack of characteristic modesty indicate that the area of the brain most likely to have been injured in this patient is the orbitofrontal cortex. Lesions of this brain area result in disinhibition, inappropriate behavior, and poor judgment. In contrast, lesions of the dorsolateral convexity of the frontal lobe result in decreased executive functioning (e.g., motivation, concentration, and attention). The hypothalamus is associated with homeostatic mechanisms and the reticular system with consciousness and sleep. Damage to the amygdala results in decreased, not increased, aggression. The nucleus basalis of Meynert is a site of Ach production; its damage could result in deficits in intellectual functioning.

18. A physician would like to prescribe an antidepressant to treat her 24-year-old male patient who has bulimia. Which of the following agents should be avoided in this patient? (A) Desipramine (B) Fluoxetine (C) Bupropion (D) Tranylcypromine (E) Paroxetine

The answer is C. Bupropion is contraindicated in eating disorder patients who also purge because it can lower the seizure threshold. The only antidepressant which is FDA approved for patients with bulimia nervosa is fluoxetine.

21. When a physician examines a 17-year- old high school student, she notes that he has erythema of the nose. During the interview, the student seems withdrawn and sad. (A) Alcohol (B) Secobarbital (C) Cocaine (D) Methylphenidate (E) Caffeine (F) Diazepam (G) Heroin (H) Marijuana (I) Nicotine (J) Phencyclidine (PCP) (K) Lysergic acid diethylamide (LSD)

The answer is C. Erythema of the nose is a result of snorting cocaine, and depressed mood is seen in withdrawal from the drug.

8. A person who uses illegal drugs is most likely to be in which of the following age groups? (A) 10-15 years (B) 15-18 years (C) 18-25 years (D) 25-35 years (E) 35-45 years

The answer is C. Illegal drug use is most common in people 18-25 years of age.

3. Which of the following structures of the mind work on an unconscious level? (A) The id only (B) The id and the ego only (C) The id, ego, and superego (D) The ego and superego only (E) Not the id, ego, or superego

The answer is C. In Freud's structural theory, the mind is divided into the id, ego, and superego. The id operates completely on an unconscious level, while the ego and superego operate partly on an unconscious and partly on preconscious and conscious levels.

14. In the United States, the group in which smoking currently shows the largest increase is (A) teenaged males (B) middle-aged males (C) teenaged females (D) middle-aged females (E) elderly females

The answer is C. In the United States, the group in which smoking currently shows the largest increase is teenaged females.

4. In Alzheimer disease patients, the major effect on neurotransmitter systems of tacrine, donepezil, rivastigmine, and galantamine is to (A) increase dopamine availability (B) decrease dopamine availability (C) increase Ach availability (D) decrease Ach availability (E) decrease serotonin availability

The answer is C. Low levels of Ach are associated with the symptoms of Alzheimer disease. Tacrine, donepezil, rivastigmine, and galantamine are acetylcholinesterase inhibitors (i.e., they block the breakdown of Ach, increasing its availability). These agents can thus be effective in slowing down the progression of the illness. They do not restore the function the patient has already lost.

15. The percentage of depressed patients who seek treatment for their symptoms is about (A) 1% (B) 5% (C) 25% (D) 50% (E) 75%

The answer is C. Only about 25% of depressed patients seek treatment, although man- agement (antidepressants, psychotherapy, electroconvulsive therapy) is effective in most depressed patients.

23. A 30-year-old woman who is taking an antipsychotic medication reports that she has been experiencing fluid discharge from the nipples. Which of the following hormones is most likely to be responsible for this problem? (A) Progesterone (B) Testosterone (C) Prolactin (D) Estrogen (E) Cortisol

The answer is C. Prolactin is the hormone responsible for galactorrhea, fluid discharge from the nipples. Galactorrhea is more common with the use of low-potency antipsy- chotic agents.

3. In a clinical experiment, a 48-year-old female patient with chronic pain who, in the past, has responded to placebos is given naloxone. Shortly thereafter the patient is given an inert substance that she believes is a painkiller. After the patient receives the inert substance, her pain is most likely to: (A) increase (B) decrease (C) be unchanged (D) respond to lower doses of opioids than previously (E) fail to respond to opioids in the future

The answer is C. Since the placebo response is based in part on activation of the endogenous opioid system, it will be blocked by naloxone, and this patient's pain will be unchanged. This experiment will not necessarily affect her response to opioids in the future.

13. The parents of a 10-year-old child report that the child is still wetting the bed. The child is very upset about this because he would like to go away to summer camp but is afraid that he will wet the bed there as well. Physical examination is unremarkable and the child is otherwise developing typically for his age. Behavioral interventions such as limiting fluids before bed and the bell and pad apparatus have not been effective. At this time, which of the following is the best choice for pharmacologic management of enuresis in this child (A) Imipramine (B) Diazepam (C) Desmopressin acetate (D) Acetaminophen (E) Aspirin

The answer is C. The best choice for the pharmacologic management of bedwetting in an older child such as this is desmopressin acetate. Imipramine is also useful in managing enuresis but has more side effects. Diazepam (a benzodiazepine), used to treat anxiety, and acetaminophen and aspirin, used in the management of minor pain, are not useful in managing enuresis.

12. A 20-year-old man reports that he just found out that his mother (whom he believed had died when he was a child) has been in an institution for the past 15 years with schizophrenia. He asks what the chances are that he will develop schizophrenia over the course of his life. The most correct answer is approximately (A) 1% (B) 5% (C) 10% (D) 50% (E) 80%

The answer is C. The chance that the son (or other first-degree relative) of a person with schizophrenia will develop the disorder over the course of his life is approximately 10%.

2. A physician becomes very angry with a patient when the patient does not take his medication. The patient reminds the doctor of her rebellious son. This physician's intense reaction to the patient's behavior is most likely to be a result of (A) positive transference (B) negative transference (C) countertransference (D) dislike of the patient (E) fear of the patient

The answer is C. The doctor who becomes very angry at her patient for not taking his medication is showing a countertransference reaction. This excessive show of emotion is a result of re-experiencing feelings about her son's behavior in her relationship with the noncompliant patient. It is important for the doctor to identify this reaction because it can interfere with her medical judgment (see also answer to the TBQ). This doctor's reaction to the patient is less likely to be related to dislike or fear of the patient.

18. A 30-year-old woman who is withdrawing from heroin shows intense anxiety, increased pulse, elevated blood pressure, and a hand tremor. Her symptoms improve when she is given clonidine, an alpha-2- adrenergic receptor agonist. The area(s) of the brain most likely to be involved in the improvement in this patient's symptoms is (are) the (A) right parietal lobe (B) basal ganglia (C) locus ceruleus (D) raphe nuclei (E) amygdala (F) substantia nigra

The answer is C. The effectiveness of clonidine in treating withdrawal symptoms asso- ciated with the use of opiates and sedatives is believed to be due to its action on alpha- 2-adrenergic receptors, for example, reducing the firing rate of noradrenergic neurons, most of which are located in the locus ceruleus.

19. A 45-year-old man with bipolar disorder tells his doctor that he has remarried and would like to have a child with his new wife. He is concerned because the 19-year-old daughter that he had with his first wife has just been diagnosed with bipolar disorder. Neither of the patient's wives has bipolar disorder. What is the probability that this patient will have another child with bipolar disorder? (A) 1% (B) 10% (C) 20% (D) 50% (E) 70%

The answer is C. The likelihood that this man with bipolar disorder will have a child with bipolar disorder is about 20%. The fact that his older child has bipolar disorder is not relevant to the chances that his next child will have the disorder

23. The mother of a 4-year-old child with diabetes takes the child to the pediatrician to "be checked" at least 3 times per week. She watches the child at all times and does not let him play outside. She also measures and remeasures his food portions three times at every meal. The mother understands that this behavior is excessive but states that she is unable to stop doing it. The most appropriate pharmacological treatment for this mother is (A) diazepam (B) buspirone (C) clomipramine (D) haloperidol (E) propranolol

The answer is C. The need to check and recheck the child's portions and repeatedly take him to the doctor indicates that, as in Question 22 and the TBQ, this patient is showing symptoms of OCD. The fact that she knows that her behavior is excessive ("insight") is typical of patients with OCD. As noted in Answer 22, the most effective long-term management for OCD is an antidepressant such as clomipramine.

21. A 12-year-old boy is admitted to the hospital with a diagnosis of "pain of unknown origin." His parents tell the physician that the child has complained about pain in his legs for about 1 month. Neurologic and orthopedic examinations fail to identify any pathology. The history reveals that the child was hospitalized on two previous occasions for other pain symptoms for which no cause was found. After 4 days in the hospital, the nurse reports that the child shows little evidence of pain and seems "remarkably content." She also reports that she found a medical textbook in the boy's bedside table with a bookmark in the section entitled "skeletal pain of unknown origin." Which of the following best describes symptom production and motivation in this case? (A) Symptom production conscious, motivation primarily conscious (B) Symptom production unconscious, motivation primarily conscious (C) Symptom production conscious, motivation primarily unconscious (D) Symptom production unconscious, motivation primarily unconscious

The answer is C. This clinical presentation is an example of factitious disorder (note: Most psychiatric diagnoses disorders can also be made in children). In contrast to patients with somatoform disorders who really believe that they are ill, patients with factitious disorder are conscious of the fact that they are feigning their illness. Pain is one of the most commonly feigned symptoms and this patient's nighttime reading is providing him with specific knowledge of how to feign the symptoms realistically. Although he is consciously producing his symptoms, this boy is not receiving tangible benefit for his behavior. Thus, in contrast to individuals who are consciously feigning illness for obvious gain, that is, malingering, the motivation for this patient's pain-faking behavior is primarily unconscious.

16. A 25-year-old man masturbates by rubbing against unsuspecting women in crowded buses. This man is showing which of the following paraphilias? (A) Fetishism (B) Exhibitionism (C) Frotteurism (D) Voyeurism (E) Sexual masochism

The answer is C. This man who masturbates by rubbing against women in crowded buses is exhibiting frotteurism. Exhibitionism involves a sexual preference for revealing one's genitals to unsuspecting persons so that they will be shocked. Fetishism is a sexual preference for inanimate objects. Sexual masochism is a preference for receiving physical pain or humiliation. Voyeurism is a preference for secretly watching people undressing or engaging in sexual activity.

24. A 22-year-old medical student is unable to stop studying until she has memorized the entire set of notes for each of her courses. Making comprehensive lists of all the subjects she must study also takes up her study time. Because of this, she is constantly behind in her work and in danger of failing her courses. (A) Borderline personality disorder (B) Histrionic personality disorder (C) Obsessive-compulsive personality disorder (D) Avoidant personality disorder (E) Antisocial personality disorder (F) Dependent personality disorder (G) Dissociative identity disorder (H) Paranoid personality disorder (I) Passive-aggressive personality disorder (J) Narcissistic personality disorder (K) Schizotypal personality disorder (L) Schizoid personality disorder

The answer is C. This medical student, who must constantly make lists and review and memorize her notes, is showing evidence of obsessive-compulsive personality disorder. This behavior is ultimately inefficient and has resulted in her academic problems.

A 36-year-old patient with schizophrenia tells the physician that the government has been listening in on all of his phone conversations for the past year. 5. This symptom indicates that the patient is most likely to have which of the following types of schizophrenia? (A) Catatonic (B) Disorganized (C) Paranoid (D) Residual (E) Undifferentiated

The answer is C. This patient is most likely to have paranoid schizophrenia, which is characterized by delusions of persecution (see also answer to Question 4).

15. A 69-year-old former bank president cannot tell you the name of the current president and has difficulty identifying the woman sitting next to him (his wife). He began having memory problems 3 years ago. The area(s) of the brain most likely to be affected in this patient is (are) the (A) right parietal lobe (B) basal ganglia (C) hippocampus (D) reticular system (E) amygdala (F) left frontal lobe

The answer is C. This patient is showing evidence of Alzheimer disease. Of the listed brain areas, the major one implicated in Alzheimer disease is the hippocampus.

11. A 28-year-old man comes in complaining of headaches and a variety of other aches and pains that have been present for the past 6 months. He denies that he is sad or hopeless. After a 4-week trial of antidepressant medication, the patient's physical complaints have disappeared. The most appropriate diagnosis for this patient is (A) dysthymic disorder (B) major depressive disorder (C) masked depression (D) hypochondriasis (E) cyclothymic disorder (F) malingering (G) bipolar disorder

The answer is C. This patient's physical complaints (i.e., headaches and other aches and pains) were relieved by antidepressant medication. This indicates that these symp- toms were manifestations of masked (hidden) depression rather than hypochondriasis. There is no evidence in this patient of bipolar disorder, dysthymic disorder, cyclothymic disorder, or malingering (see also answer to Question 10).

10. A 28-year-old stockbroker who is married and has two children usually dresses conservatively. She receives a letter containing a recent photograph of herself in a skimpy black leather outfit. She does not remember the man who signed the letter, or posing for the photograph. This woman is showing evidence of (A) dissociative amnesia (B) dissociative fugue (C) dissociative identity disorder (D) depersonalization disorder (E) schizophrenia

The answer is C. This stockbroker is showing evidence of dissociative identity disorder (formerly multiple personality disorder). She does not remember the man who signed the letter or posing for the photograph because these events occurred when she was showing another personality. Dissociative amnesia involves a failure to remember important information about oneself, and dissociative fugue is amnesia combined with sudden wandering from home and taking on a different identity. Depersonalization disorder is a persistent feeling of detachment from one's own body, the social situation, or the environment (derealization) (and see also answer to Question 9).

15. A physician conducts a yearly physical examination on a typical 85-year-old patient. Which of the following mental characteristics is the doctor most likely to see in this patient? (A) Impaired consciousness (B) Abnormal level of arousal (C) Minor forgetfulness (D) Psychosis (E) Depression

The answer is C. This typical 85-year-old patient is likely to show minor forgetfulness, such as forgetting new names and phone numbers. Impaired consciousness, psychosis, and abnormal level of arousal are seen in delirium, which is associated with a variety of physical illnesses. As in younger people, in the elderly depression is an illness (see Chapter 12), not a natural consequence of typical aging.

19. What is the antidepressant agent most likely to cause persistent erections (priapism) in a 40-year-old male patient? (A) Venlafaxine (B) Tranylcypromine (C) Trazodone (D) Doxepin (E) Amoxapine (F) Fluoxetine (G) Nortriptyline (H) Imipramine

The answer is C. Trazodone is the agent most likely to cause priapism in this patient.

21. Uterine contractions mainly occur in which stage of the sexual response cycle? (A) Excitement (B) Plateau (C) Orgasm (D) Resolution

The answer is C. Uterine contractions occur mainly during the orgasm phase of the sexual response cycle.

13. An 18-year-old student who is about 10 pounds overweight tells her physician that she has decided to go on a low carbohydrate diet that she read about in a book. She says that the book guarantees that people who follow the diet will lose at least 25 pounds in 3 weeks. The doctor's best statement to the patient at this time is (A) That is nonsense, you can't lose that much weight in only 3 weeks. (B) You may lose the weight but you will end up gaining back even more weight. (C) Please tell me more about the book that you read. (D) You may be showing signs of an eating disorder. (E) Many people your age have eating disorders.

The answer is C. The doctor's best statement to the patient at this time is, "Please tell me more about the book that you read." It is important to get as much information as possible from the patient before deciding on a course of action

A 9-year-old boy with normal intelligence frequently gets into trouble at school because he blurts out answers, interrupts the teacher, disturbs the other students, and cannot seem to sit still in class. He also fre- quently injures himself during play and rarely sits through an entire meal at home. His siblings say that he is "a real pest." However, the child does his schoolwork well and behaves well when he is alone with his tutor. 7. This boy is at a higher risk than other children to develop which of the following disorders? (A) Tourette disorder (B) Separation anxiety disorder (C) Bipolar disorder (D) Conduct disorder (E) Schizophrenia

The answer is D Children with ADHD are at higher risk than other children for oppositional defiant disorder and conduct disorder.

A 22-year-old man with schizophrenia who has been taking an antipsychotic for the past 3 months reports that recently he has experienced an uncomfortable sensation in his arms and legs during the day and must constantly move them. Because of this, he can sit still for only a few minutes at a time. This medication side effect is best described as 2. The antipsychotic agent that this patient is most likely to be taking is (A) risperidone (B) thioridazine (C) olanzapine (D) haloperidol (E) clozapine

The answer is D High- potency antipsychotics, such as haloperidol, are more likely to cause these neurologic side effects than low-potency agents such as thioridizine, or atypical agents, such as risp- eridone, olanzapine, and clozapine.

A 45-year-old woman says that she frequently feels "nervous" and often has an "upset stomach," which includes heartburn, indigestion, and diarrhea. She has had this problem since she was 25 years of age and notes that other family members also are "tense and nervous." 6. Of the following, the most effective long- term management for this patient is (A) alprazolam (Xanax) (B) psychotherapy (C) propranolol (Inderal) (D) buspirone (BuSpar) (E) diazepam (Valium)

The answer is D Of the choices, the most effective long-term management for this patient is buspirone because, unlike the benzodiazepines alprazolam and diazepam, it does not cause dependence or withdrawal symptoms with long-term use. The antidepressants venlafaxine and duloxetine and SSRIs also are effective for long-term management of GAD. Psychotherapy and β-blockers can be used as adjuncts to treat GAD, but are not the most effective long-term treatments.

A 39-year-old woman takes her 6-year-old son to a physician's office. She says that the child often experiences episodes of breath- ing problems and abdominal pain. The child's medical record shows many office visits and four abdominal surgical procedures, although no abnormalities were ever found. Physical examination and laboratory studies are unremarkable. When the doctor confronts the mother with the suspicion that she is fabricating the illness in the child, the mother angrily grabs the child and leaves the office immediately. 10. In this situation, what is the first thing the physician should do? (A) Take the child aside and ask him how he feels. (B) Call a pediatric pulmonologist to determine the cause of the dyspnea. (C) Call a pediatric gastroenterologist to determine the cause of the abdominal pain. (D) Notify the appropriate state social service agency to report the physician's suspicions. (E) Wait until the child's next visit before taking any action.

The answer is D The first thing the physician must do is to notify the state social service agency since factitious disorder by proxy is a form of child abuse. Waiting until the child's next visit before acting could result in the child's further injury or even death. Calling in specialists may be appropriate after the physician reports his suspicions to the state. It is not appropriate to take the child aside and ask him how he really feels. He probably is not aware of his mother's behavior.

A 23-year-old medical student comes to the emergency room with elevated heart rate, sweating, and shortness of breath. The student is convinced that she is having an asthma attack and that she will suffocate. The symptoms started suddenly during a car ride to school. The student has had episodes such as this on at least three previous occa- sions over the past 2 weeks and now is afraid to leave the house even to go to school. She has no history of asthma and, other than an increased pulse rate, physical findings are unremarkable. 3. The neural mechanism most closely involved in the etiology of this patient's symptoms is (A) nucleus accumbens hyposensitivity (B) ventral tegmental hypersensitivity (C) ventral tegmental hyposensitivity (D) locus ceruleus hypersensitivity (E) peripheral autonomic hypersensitivity

The answer is D The neural etiology most closely involved in panic disorder with agoraphobia is hypersensitivity of the locus ceruleus.

A 54-year-old woman with schizophrenia who has been taking a high-potency antipsychotic agent for the past 5 years has begun to show involuntary chewing and lip-smacking movements. 3. This sign indicates that the patient is experiencing a side effect of antipsychotic medication known as (A) restless legs syndrome (B) neuroleptic malignant syndrome (C) akathisia (D) tardive dyskinesia (E) acute dystonia (F) pseudoparkinsonism

The answer is D These involuntary chewing and lip-smacking move- ments indicate that the patient has developed tardive dyskinesia, a serious side effect of treatment with antipsychotic medication

Concerned parents of a 7-year-old boy bring their child to the pediatrician for evaluation. They note that ever since he was an infant, their son has never wanted to be held, cries whenever he is bathed, and becomes very upset when his daily routine is changed in any way. Although the child cannot yet read, his parents remark that he can identify the state of origin of any car license plate and almost exclusively plays with replicas of car license plates. The child's speaks in complete sentences and has a good vocabulary, but his behavior seems odd and he does not make eye contact when spoken to. Medical evaluation is unremarkable. 10. As an adolescent, this boy is likely to have the most difficulty in which of the following areas? (A) Paying attention in school (B) Concentrating on relevant stimuli (C) Caring for pets (D) Making friends (E) Controlling his activity level

The answer is D This child who does not want to be held, cries when his environment is changed (e.g., when bathed), and does not make eye contact is likely to have ASD. Children with ASD have great difficulty with interpersonal rela- tions. Problems with attention and concentration are more characteristic of ADHD. Chil- dren with conduct disorder tend to have poor self-control and to break societal rules. Hyperactivity may be present but is not specifically associated with ASD

Since the age of 8, a 15-year-old girl with normal intelligence and interactive skills has shown a number of repetitive motor movements. She recently has begun to have outbursts in which she curses and shrieks. When asked if she can control the vocalizations and movements she says, "For a short time only; it is like holding your breath—eventually you have to let it out." Medical evaluation is unremarkable. 1. This child is showing evidence of (A) autism spectrum disorders (ASD) (B) Rett disorder (C) attention deficit hyperactivity disorder (ADHD) (D) Tourette disorder (E) selective mutism

The answer is D This girl is most likely to have Tourette disorder, a chronic neurologic condition with behavioral manifestations such as unwanted motor activity and vocalizations. The vocalizations and motor tics can be controlled only briefly and then they must be expressed. ASD and Rett disorder are pervasive developmental disorders of childhood, that are characterized by abnormal social interaction and speech. ADHD involves normal development of speech and social interaction but difficulty pay- ing attention or sitting still. Selective mutism involves voluntary absence or decrease in speaking in social situations.

A 21-year-old ballet dancer, who is 5 feet 7 inches tall and has weighted 95 pounds (BMI = 14.5) for the past year, tells the doctor that she needs to lose another 15 pounds to pursue a career in dance. Her mood appears good. Findings on physical examination are normal except for excessive growth of downy body hair. She reports that she has not menstruated in more than 3 years. 16. Which of the following is most likely to characterize this female? (A) Lack of interest in preparing food (B) Embarrassment about her appearance (C) Lack of appetite (D) Conflict with her mother (E) Poor school performance

The answer is D This woman is already underweight yet wants to lose more weight, and she has developed lanugo (growth of downy body hair) and amenorrhea (absence of menses). These findings indicate that she has anorexia ner- vosa. Since dancers and gymnasts often must be small and slim, these activities are closely associated with the development of anorexia nervosa. Anorexia is also charac- terized by family conflicts, particularly with the mother; normal appetite; high interest in food and cooking; low sexual interest; good school performance; and excessive exer- cising.

A 34-year-old man has been taking fluoxetine for treatment of depression for the past 4 months. His mood is now normal but he reports that he is having sexual problems. 8. Which of the following sexual dysfunctions is this man most likely to report? (A) Primary erectile disorder (B) Secondary erectile disorder (C) Premature ejaculation (D) Delayed orgasm (E) Dyspareunia

The answer is D While they may be associated with loss of libido and erectile disorder, fluoxetine and other selective serotonin reuptake inhibitors (SSRIs) are more likely to cause delayed or absent orgasm (orgasmic disorder). That is why the SSRIs are useful in managing premature ejaculation. Dyspareunia is not associated spe- cifically with SSRI treatment.

15. A 31-year-old lawyer is transferred to the emergency room after collapsing at a party. He complained of chest pain, and an electrocardiogram demonstrated ventricular fibrillation, for which he received cardioversion. A primary survey of the patient showed little trauma, with the exception of a perforated nasal septum. A close friend accompanied the patient and confides in you that the patient was "doing" an illicit substance at the party. Which of the following is the most likely? (A) Phencyclidine (PCP) (B) γ-Hydroxybutyric acid (GHB) (C) Lysergic acid diethylamide (LSD) (D) Cocaine (E) Marijuana

The answer is D. Cocaine is cardiotoxic and can cause arrhythmias that can be life- threatening. These effects are even more likely when alcohol is also consumed. Cocaine causes vasoconstriction, and snorting the drug causes necrosis and eventual perforation of the nasal septum. γ-Hydroxybutyric acid (GHB) is a barbiturate that is used as a "date rape" drug. Lysergic acid diethylamide (LSD) causes increased sensory awareness, perceptual distortions, and altered consciousness. Phencyclidine (PCP) can cause euphoria, hallucinations, an increased sense of isolation and loneliness, and increased aggression. Marijuana causes euphoria, laughter, a loss of time perception, and increased introspection.

10. A physician discovers that his 28-year- old female patient is abusing cocaine. Which of the following can the doctor expect to see in this patient? (A) Severe physical signs of withdrawal (B) Little psychological craving in withdrawal (C) Euphoria lasting 3-4 days (D) Delusions (E) Sedation with use

The answer is D. Delusions and other symptoms of psychosis are seen with the use of cocaine. The intense euphoria produced by cocaine lasts only about 1 hour. Severe psychological craving for the drug peaks 2-4 days after the last dose, although there may be few physiologic signs of withdrawal. Cocaine intoxication is characterized by agitation and irritability, not sedation.

13. A 28-year-old alcoholic woman learns that she is pregnant after missing her last two menstrual periods. She received poor prenatal care, missing many of her appointments. The neonatologist that cared for her child at birth learned that the mother did not refrain from her normal heavy alcohol binges throughout her pregnancy. Which of the following is the most likely consequence of such abuse to the child during pregnancy? (A) Abruptio placentae and learning difficulties as the child ages (B) Spina bifida and orofacial defects (C) Fetal hemorrhage and defects in fetal bone formation (D) Microcephaly, retarded growth, and congenital heart defects (E) Growth retardation, microcephaly, and craniofacial abnormalities

The answer is D. Fetal alcohol syndrome is a leading cause of congenital abnormalities, especially microcephaly, growth retardation, and congenital heart defects. Abruptio placentae and learning difficulties are more typical with maternal use of cocaine during pregnancy. Spina bifida and orofacial defects are associated with valproic acid use during pregnancy, and the anticonvulsant phenytoin is associated with fetal growth retardation, microcephaly, and craniofacial abnormalities. Fetal hemorrhage and bone malformation are a consequence of warfarin use during pregnancy.

4. The above patient returns for follow-up 6 months later. He has joined a health club and has lost several pounds and notes "feeling better mentally." However, upon questioning he still has not returned to the activities he once enjoyed and is still not sleeping or eating well. The psychiatrist rec- ommends increasing the dose of his selective serotonin reuptake inhibitor (SSRI). The patient reluctantly admits that he has not been taking his medication because of some of the side effects. Which one is likely to be the most bothersome? (A) Weight gain (B) Tachycardia (C) Headache (D) Sexual dysfunction (E) Tremor

The answer is D. Sexual dysfunction is a common complaint with selective serotonin reuptake inhibitors (SSRIs), occurring in up to 40% of all patients, and a leading cause of noncompliance. Weight loss is usually experienced initially with SSRIs, and some persons may eventu- ally gain weight. Headache is associated with SSRIs, although it is often transient. Tremor and tachycardia are side effects that are more typical with tricyclic antidepressants (TCAs).

8. A 30-year-old woman tells the physician that she must drive the route she takes home from work each day at least three times to be sure that she did not hit an animal in the road. Of the following, the most appropriate long-term pharmacological management for this patient is (A) a high-potency antipsychotic agent (B) an anticholinergic agent (C) an anti anxiety agent (D) an antidepressant agent (E) lithium

The answer is D. The most effective pharmacological treatment for this patient who has obsessive-compulsive disorder is an antidepressant, particularly a selective serotonin reuptake inhibitor (see Chapter 13). Antipsychotics, antianxiety agents, and lithium are less appropriate than an antidepressant for this patient.

20. A 22-year-old man is brought into the emergency room by the police. The police- man tells the physician that the man was caught while attempting to rob a bank. When the police told him to freeze and drop his gun, the man dropped to the floor and could not speak, but remained conscious. When the doctor attempts to interview him, the patient repeatedly falls asleep. The history reveals that the patient's brother has narcolepsy. Which of the following best fits this clinical picture? (A) A sleep disorder (B) A seizure disorder (C) A somatoform disorder (D) Malingering (E) An endocrine disorder (F) A factitious disorder

The answer is D. When there is financial or other obvious gain to be obtained from an illness, the possibility that the person is malingering must be considered. In this case, a man who has committed a crime is feigning symptoms of narcolepsy to avoid prosecution. Knowledge of the details of his brother's illness has taught him how to feign the cataplexy (sudden loss of motor control) and daytime sleepiness associated with narcolepsy

2. A 43-year-old man comes to the emergency department of a large hospital. He is very anxious and complains of abdominal cramps and diarrhea. The physician observes intense flushing of the man's skin. In this patient, a 24-hour urine study is most likely to reveal elevated levels of (A) acetylcholine (B) 5-hydroxyindoleaceticacid (5-HIAA) (C) porphobilinogen (D) vanillylmandelicacid(VMA) (E) homovanillicacid(HVA)

The answer is D. A 24-hour urine study is most likely to reveal elevated levels of VMA, a metabolite of norepinephrine. Anxiety, abdominal cramps and diarrhea, and skin flushing are symptoms of pheochromocytoma, a norepinephrine-secreting adrenal tumor. This picture is not seen with elevated levels of other neurotransmitter metabolites.

13. A 55-year-old woman was diagnosed with schizophrenia at the age of 22. If this diagnosis was appropriate, the volume of the hippocampus, the size of the cerebral ventricles, and glucose utilization in the frontal cortex of this patient are now most likely to be, respectively (A) increased, increased, increased (B) decreased, decreased, decreased (C) decreased,decreased,increased (D) decreased, increased, decreased (E) increased,decreased,increased

The answer is D. Although neuroimaging cannot be used to diagnose psychiatric disor- ders, brains of patients with schizophrenia such as this woman are likely to show decreased volume of limbic structures such as the hippocampus; increased size of cer- ebral ventricles due, in part, to brain shrinkage; and decreased glucose utilization in the frontal cortex.

14. The percentage of patients with schizophrenia who attempt suicide is approximately (A) 1% (B) 5% (C) 12% (D) 50% (E) 80%

The answer is D. Approximately 50% of patients with schizophrenia attempt suicide at some point in their lives.

17. A 28-year-old male patient is brought to the emergency room after a fight in which he attacked a man who cut into his line at the supermarket checkout. In the emergency room he remains assaultive and combative. The body fluids of this patient are most likely to show (A) increased 3-methoxy-4-hydroxyphenylglycol (MHPG) (B) decreased MHPG (C) increased 5-hydroxyindole acetic acid (5-HIAA) (D) decreased 5-HIAA (E) decreased homovanillic acid (HVA)

The answer is D. Assaultive, impulsive, aggressive behavior like that seen in this 28-year-old male patient is associated with decreased levels of serotonin in the brain. Levels of 5-HIAA (5-hydroxyindoleacetic acid), the major metabolite of serotonin, have been shown to be decreased in the body fluids of violent, aggressive, impulsive individuals as well as depressed individuals. MHPG (3-methoxy-4-hydroxyphenylglycol), a metabolite of norepinephrine, is decreased in severe depression, while homovanillic acid (HVA), a metabolite of dopamine, is decreased in Parkinson disease and depression.

A 26-year-old medical student is brought to the emergency department by her husband. The husband tells the doctor that his wife has shown odd behavior ever since failing an exam 2 weeks ago. In particular, she tells him that people are trying to poison her. The woman has no prior psychiatric history, and physical examination and laboratory results are unremarkable. 2. The patient's belief that people are trying to poison her is an example of (A) an illusion (B) a neologism (C) a hallucination (D) a delusion (E) an idea of reference

The answer is D. Believing that you are being poisoned is a delusion, that is, a false belief. A hallucination is a false perception; an illusion is a misperception of real external stimuli; an idea of reference is the false belief of being referred to by others; and a neologism is a new, invented word. All of these phenomena can be seen in patients exhibiting psychotic symptoms no matter what the cause.

8. A 5-year-old child is admitted to the hospital with a low-grade fever and a persistent cough that has resulted in vomiting episodes following prolonged coughing spells. His throat culture is negative, his fever has resolved, and all that is left is a slight cough. He is discharged from the hospital by the pediatrician who recommends an over-the-counter opioid antitussive. Which of the following does he recommend? (A) Tramadol (B) Diphenoxylate (C) Loperamide (D) Dextromethorphan (E) Naloxone

The answer is D. Dextromethorphan is an opioid isomer available in over-the-counter cough remedies. It has no analgesic properties and limited abuse potential at recommended doses. Tramadol is a weak μ-opioid-receptor agonist, which also blocks serotonin and norepineph- rine uptake and is used for neuropathic pain. Diphenoxylate is an opioid that, combined with atropine, is taken orally to treat diarrhea. Loperamide is an opioid that does not cross the blood-brain barrier and is also used for the treatment of diarrhea. Naloxone is an opioid antagonist used to reverse opioid overdose.

1. A female patient who is hospitalized after making a suicide attempt because her doctor did not respond to her offer to friend him on Facebook states that all female doctors are good but all male doctors are incompetent. The best explanation for this statement by the patient is (A) prejudice (B) lack of basic trust (C) chauvinism (D) splitting (E) bias

The answer is D. Dividing people or situations into categories of good and bad charac- terizes the defense mechanism of splitting. Splitting is commonly used by people with the borderline personality disorder (see Chapter 14).

4. A 65-year-old female patient has had a stroke affecting the left hemisphere of her brain. Which of the following functions is most likely to be affected by the stroke? (A) Perception (B) Musical ability (C) Spatial relations (D) Language (E) Artisticability

The answer is D. Dominance for language in both right-handed and left-handed people is usually in the left hemisphere of the brain. Perception, musical ability, artistic ability, and spatial relations primarily are functions of the right side of the brain.

6. A 22-year-old student tells the doctor that he has been using "speed" nightly. Which of the following effects of the drug is the student most likely to experience when he is using? (A) Increased fatigue (B) Decreased pain threshold (C) Increased appetite (D) Decreased appetite (E) Decreased libido

The answer is D. Like other stimulant drugs, amphetamines like "speed" reduce appetite; use can thus result in decreased body weight. *Amphetamines also decrease fatigue, increase pain threshold, and increase libido.*

20. Which stage of the sexual response cycle shows the greatest difference between men and women? (A) Excitement (B) Plateau (C) Orgasm (D) Resolution

The answer is D. Resolution shows the greatest difference between men and women. Men have a resting (refractory) period after orgasm when restimulation is not possible. Women are less likely than men to have a refractory period.

6. A 23-year-old patient shows side effects such as sedation, increased appetite, and weight gain while being treated with antipsychotic medication. Of the following, the mechanism most closely associated with these effects is (A) blockade of serotonin receptors (B) blockade of dopamine receptors (C) blockade of norepinephrine receptors (D) blockade of histamine receptors (E) decreased availability of serotonin

The answer is D. Sedation, increased appetite, and weight gain are side effects of treat- ment with certain antipsychotic agents. The mechanism most closely associated with these side effects is blockade of histamine receptors since these antipsychotics are not specific for dopamine blockade. Blockade of dopamine receptors by these antipsychotic medications is associated with side effects such as Parkinsonism-like symptoms and elevated prolactin levels.

12. A 35-year-old female patient reports that she has difficulty sleeping ever since she sustained a concussion in a subway accident. The area(s) of the brain most likely to be affected in this patient is (are) the (A) right parietal lobe (B) basal ganglia (C) hippocampus (D) reticular system (E) amygdala (F) left frontal lobe

The answer is D. Sleep-arousal mechanisms are affected by damage to the reticular system.

5. A primary care physician notices that many of her patients use statements like "I can't stop smoking because I'll gain weight," or "when I'm sick, I only want to eat junk food." Statements like these (A) produce conflict in the conscious mind (B) are conscious mental techniques (C) increase anxiety (D) are examples of the use of defense mechanisms (E) decrease a patient's sense of self-esteem

The answer is D. Statements such as "I can't stop smoking because I'll gain weight," or "when I'm sick, I only want to eat junk food" are examples of the defense mechanisms of rationalization and regression, respectively. In rationalization, a person distorts her perception of an event so that its negative outcome seems reasonable, for example, because she feels unable to stop smoking, this patient claims (and so she reasonably feels) that gaining weight is worse than smoking, a life-threatening habit. In regression, ill patients revert to behavior patterns like those seen in someone of a younger age (e.g., eating junk food, crying). Defense mechanisms such as these are unconscious mental techniques that decrease anxiety and help people to maintain a sense of equilibrium and self-esteem.

10. The best choice of antianxiety agent for a 40-year-old woman with generalized anxiety disorder and a history of benzodiazepine abuse is (A) zolpidem (B) flurazepam (C) clonazepam (D) buspirone (E) chlordiazepoxide (F) bupropion

The answer is D. The best choice of antianxiety agent for a 40-year-old patient with generalized anxiety disorder and a history of BZ abuse is buspirone, a non-benzodi- azepine with very low abuse potential. Benzodiazepines such as flurazepam, clonazepam, and chlordiazepoxide have higher abuse potential than buspirone. Bupropion is an antidepressant, which is also used for smoking cessation. Zolpidem is a non- benzodiazepine sleep agent.

17. A 43-year-old man with a 5-year history of HIV tells his physician that he has been smoking marijuana a few times a day to treat his symptoms of nausea and lack of appe- tite. To obtain the marijuana, the patient notes that he grows it in his backyard. The doctor's best response to this patient's revelation is (A) "I am sorry but growing or using marijuana is illegal and I must notify the police" (B) "I have read about other patients grow- ing marijuana" (C) "Are you aware that marijuana can cause respiratory problems?" (D) "There are a number of medications that I can prescribe to help alleviate your nausea and lack of appetite in place of marijuana" (E) "Do you think that using marijuana has negative long-term effects?"

The answer is D. The best response to this patient's revelation about growing and using marijuana is to recommend effective but safer substitutes, for example, pre- scription medications to treat his nausea and lack of appetite. It is neither appropriate nor necessary for a physician to report the patient's actions to the police. Also, this HIV-positive patient is likely to be more concerned about feeling ill in the short-term than long-term consequences of marijuana use such as respiratory problems.

5. Which of the following two structural entities connect the cerebral hemispheres? (A) Basal ganglia and anterior commissure (B) Anterior commissure and reticular system (C) Reticular system and corpus callosum (D) Hippocampal commissure and corpus callosum (E) Amygdala and habenular commissure

The answer is D. The corpus callosum and the hippocampal, habenular, and anterior commissures connect the two hemispheres of the brain. The basal ganglia, reticular system, and amygdala do not have this function.

A 29-year-old man comes to the emergency department complaining of stomach cramps, agitation, severe muscle aches, and diarrhea. Physical examination reveals that the patient is sweating, has dilated pupils, a fever, and a runny nose, and shows goose bumps on his skin. 1. Of the following, the most likely cause of this picture is (A) alcohol use (B) alcohol withdrawal (C) heroin use (D) heroin withdrawal (E) amphetamine withdrawal

The answer is D. The most likely cause of this patient's symptoms of sweating, muscle aches, stomach cramps, diarrhea, fever, runny nose, goose bumps, yawning, and dilated pupils is heroin withdrawal. While alcohol withdrawal may be associated with pupil dilation, alcohol use and withdrawal and amphetamine withdrawal are less likely to cause this constellation of symptoms.

4. A physician is doing an employment physical on a 40-year-old male patient. The physician suspects that the patient has a problem with alcohol. The next step that the physician should take is to (A) check his liver function (B) ask him if he has a problem with alcohol (C) call his previous employer for information (D) ask him the CAGE questions (E) check for the stigmata of alcoholism (e.g., stria, broken blood vessels on the nose)

The answer is D. The next step in management is for the physician to ask this patient the CAGE questions. Positive answers to any two of these questions or to the last one alone indicate that he has a problem with alcohol. Patients with such problems typically use denial as a defense mechanism and so rarely believe or admit that they have a problem with alcohol. Liver function problems or presence of the stigmata of alcoholism (e.g., stria, broken blood vessels on the nose) do not necessarily indicate the patient currently has a problem with alcohol. It is inappropriate for the doctor to call the previous employer for information.

1. A 65-year-old woman, who was diagnosed with advanced lung cancer 3 months ago, has lost 18 pounds, wakes frequently during the night, and has very little energy. Over the past month she has been preoccupied with feelings of guilt about "people I have hurt in my life" and expresses concern that she will die in pain. The sign or symptom most likely to indicate that this patient is experiencing a major depressive episode rather than a nor- mal reaction to life-limiting illness is (A) weight loss (B) decreased energy (C) difficulty sleeping (D) preoccupation with feelings of guilt (E) concern about dying pain

The answer is D. The sign or symptom most likely to indicate that this patient is experiencing a major depressive episode rather than a normal reaction to serious illness is her preoccupation with feelings of guilt. Such feelings are more characteristic of depression than sadness about being very ill. The other symptoms that the patient shows (e.g., weight loss, decreased energy, and sleep problems) are characteristic of patients with advanced cancer. Fear of dying in pain is realistic and commonly seen in patients with life-limiting illnesses.

9. A 15-year-old steals from family members and friends. When no one is watching, he also tortures the family cat. Which aspect of the mind is deficient in this teenager? (A) The unconscious mind (B) The preconscious mind (C) The conscious mind (D) The superego (E) The ego

The answer is D. The superego is associated with moral values and conscience, and controls impulses of the id. This teenager who steals from family members and friends and tortures the family cat is showing deficiencies in his superego. Children and ado- lescents under age 18 years, who have poor superego development, may be diagnosed with conduct disorder

26. A 20-year-old female college student who was unable to answer a teacher's question in class drops out of school the next day. (A) Borderline personality disorder (B) Histrionic personality disorder (C) Obsessive-compulsive personality disorder (D) Avoidant personality disorder (E) Antisocial personality disorder (F) Dependent personality disorder (G) Dissociative identity disorder (H) Paranoid personality disorder (I) Passive-aggressive personality disorder (J) Narcissistic personality disorder (K) Schizotypal personality disorder (L) Schizoid personality disorder

The answer is D. This 20-year-old female college student shows evidence of avoidant personality disorder. She is so overwhelmed by what she perceives as criticism and rejection that she drops out of school rather than face her teacher and classmates again.

18. A 35-year-old man is brought to the emergency department confused and anxious. The man reports that someone is trying to kill him but he does not know who the person is. Initial physical examination reveals elevated heart and respiration rates. While in the emergency room the patient has a seizure and then develops life-threatening cardiovascular symptoms. The drug that this patient is most likely to be withdrawing from is (A) phencyclidine (PCP) (B) lysergic acid diethylamide (LSD) (C) heroin (D) secobarbital (E) marijuana

The answer is D. This 35-year-old patient is most likely to be withdrawing from secobarbital, a barbiturate. Barbiturate withdrawal symptoms appear about 12-20 hours after the last dose and include anxiety, elevated heart and respiration rates, psychotic symptoms (e.g., the belief that someone is trying to kill him), confusion, and seizures, and can be associated with life-threatening cardiovascular symptoms. There are few physical withdrawal symptoms associated with marijuana, phencyclidine (PCP), or lysergic acid diethylamide (LSD), and those associated with heroin are uncomfortable but rarely physically dangerous.

18. A 30-year-old financial consultant tells her doctor that over the past 5 years she has felt "down" most of the time. She relates that when colleagues ask her to dinner or to a get-together she usually says "yes" but then rarely feels like going when the time comes and does not have a good time when she does go. There are no significant physical findings. While the patient denies suicidality, she notes that she never feels really excited or happy about anything. The best diagnosis for this patient at this time is (A) major depressive disorder (B) bipolar I disorder (C) bipolar II disorder (D) dysthymic disorder (E) cyclothymic disorder (F) substance-induced mood disorder (G) mood disorder due to a general medical condition

The answer is D. This best diagnosis for this patient is dysthymic disorder. The patient has had a low mood for years, but although she is never really happy or excited about what should be pleasant experiences, because she is functional. For example, she is working and is not suicidal, it is less likely that she has major depressive disorder

9. A 9-year-old boy with normal intelligence has a history of fighting with other children and catching and torturing birds, squirrels, and rabbits. When asked why he engages in this behavior, he says, "It's just fun." Developmental history and medical examination are unremarkable. The best explanation of this child's behavior is (A) oppositional defiant disorder (B) ADHD (C) social difficulties in the family (D) conduct disorder (E) normal age-appropriatebehavior

The answer is D. This child is showing evidence of conduct disorder. Children with this disorder have little or no concern for others or for animals (e.g., this child finds torturing animals "fun")

16. A 15-year-old girl is brought to the emergency room after ingesting 20 acetaminophen tablets. She tells the physician that she tried to commit suicide because she was not admitted to an honors English class. The girl is the president of her grade in school and always tries to be perfect. The most important factor in whether this girl tries to kill herself again is (A) that she is female (B) the method of the suicide attempt (C) that she has major depressive disorder (D) that she tried to commit suicide once (E) her need to be perfect

The answer is D. This girl shows a number of risk factors for depression and attempted suicide, including female sex and her excessive need to be perfect. However, the most important factor in whether she tries to kill herself again is that she tried to commit suicide once. Taking pills such as aspirin or acetaminophen is less lethal than other methods, but young people such as this teenager may not know this. Thus, this girl has made a serious suicide attempt.

11. A 35-year-old man who lives in a group home says that his roommates are spying on him by listening to him through the electrical outlets. For this reason, he has changed roommates a number of times over the last 5 years. He dresses strangely, is dirty with unkempt hair, and seems preoccupied. He reports that he has trouble paying attention to the doctor's questions because "I am listening to my leader giving me instructions in my head." Neuropsychological evaluation of this patient when he is not hearing voices is most likely to reveal (A) memory impairment (B) inability to state his name (C) mental retardation (D) frontal lobe dysfunction (E) lack of orientation to place

The answer is D. This man, who dresses strangely, shows poor grooming, and has para- noid delusions and auditory hallucinations over a prolonged period, is most likely to have schizophrenia. *Neuropsychological evaluation of a patient with schizophrenia is most likely to reveal frontal lobe dysfunction*. People with schizophrenia usually show intact memory; orientation to person, place, and time; and normal intelligence.

A 26-year-old medical student is brought to the emergency department by her husband. The husband tells the doctor that his wife has shown odd behavior ever since failing an exam 2 weeks ago. In particular, she tells him that people are trying to poison her. The woman has no prior psychiatric history, and physical examination and laboratory results are unremarkable. 1. What is the most appropriate diagnosis for this patient at this time? (A) Schizophrenia (B) Schizoaffective disorder (C) Schizophreniform disorder (D) Brief psychotic disorder (E) Delusional disorder (F) Shared psychotic disorder (G) Psychosis due to a general medical condition

The answer is D. This patient is showing evidence of brief psychotic disorder. This disorder is characterized by psychotic symptoms lasting >1 day, but *<1 month*; she has had symptoms for the past 2 weeks. Also, the stress of failing the exam is likely to be a precipitating psychosocial factor in this patient. Schizoaffective disorder is characterized by symptoms of both a mood disorder and schizophrenia, as well as psychotic symptoms that occur even in the absence of mood symptoms, and lifelong social and occupational impairment. In *schizophrenia*, psychotic and residual symptoms last *6 months*, and there is lifelong social and occupational impairment. Schizophreniform disorder is characterized by psychotic and residual symptoms lasting 1-6 months. In delusional disorder, which often lasts for years, there is a fixed, *non-bizarre delusional system*; few, if any, other thought disorders; and *relatively normal social and occupa- tional functioning.* In shared psychotic disorder, a person develops the same delusion as a person with delusional disorder with whom they are in a close relationship. Psychosis due to a general medical condition involves psychotic symptoms occurring as a result of physical illness

7. A 26-year-old man comes to the emergency department with elevated blood pressure, sweating, headache, and vomiting. His companion tells the physician that the patient became ill at a party where he ate pizza and drank alcoholic punch. The drug that this patient is most likely to be taking is (A) fluoxetine (B) lithium (C) nortriptyline (D) tranylcypromine (E) haloperidol

The answer is D. This patient who became ill at a pizza party is most likely to be taking tranylcypromine, a monoamine oxidase inhibitor (MAOI). These agents can cause a hypertensive crisis if certain foods (e.g., aged cheese, smoked meats, beer, and wine) are ingested. A patient who eats in an unfamiliar place (e.g., a party) may unwittingly ingest forbidden foods. This patient ate pizza that probably contained aged Parmesan cheese and drank punch that probably contained red wine. This resulted in a hypertensive crisis (e.g., elevated blood pressure, sweating, headache, and vomiting). Fluoxetine, lithium, nortriptyline, and haloperidol do not interact negatively with food.

9. A 43-year-old woman says that when she is under stress, she often feels as if she is "outside of herself" and is watching her life as though it were a play. She knows that this perception is only a feeling and that she is really living her life. This woman is showing evidence of (A) dissociative amnesia (B) dissociative fugue (C) dissociative identity disorder (D) depersonalization disorder (E) schizophrenia

The answer is D. This woman, who feels as if she is "outside of herself," watching her life as though it were a play, is showing evidence of depersonalization disorder, a persistent feeling of detachment from one's own body or the social situation. In contrast to psychotic disorders such as schizophrenia (see Chapter 11), this woman is aware that this perception is only a feeling and that she is really living her life.

13. A patient reports that his 19-year-old identical twin brother has just been diagnosed with schizophrenia and wants to know what the likelihood is that he will develop this disorder. The most correct answer is approximately (A) 1% (B) 5% (C) 10% (D) 50% (E) 80%

The answer is D. The chance that the identical twin of a person with schizophrenia will develop the disorder over the course of his or her life is approximately 50%.

A 62-year-old woman whose husband died 6 months ago tells her physician that she believes killing herself would end her suffering. Physical examination is unremarkable. 8. Analysis of neurotransmitter availability in the brain of this patient is most likely to reveal (A) increased dopamine (B) decreased histamine (C) increased acetylcholine (D) decreased acetylcholine (E) decreased serotonin

The answer is E Analysis of neurotransmitter availability in this patient is most likely to reveal decreased serotonin, commonly reflected in decreased plasma levels of its major metabolite 5-HIAA. Increased dopamine is seen in schizophrenia, and decreased ace- tylcholine is seen in Alzheimer disease.

A 25-year-old patient who has taken haloperidol for the past 2 months is brought to the hospital with a temperature of 104°F, blood pressure of 190/110, and muscular rigidity. 6. The side effect is best treated initially (A) by changing to a low-potency antipsy- chotic agent (B) with an antianxiety agent (C) with an antidepressant agent (D) with an anticonvulsant (E) by stopping the antipsychotic agent

The answer is E Neuroleptic malignant syndrome is seen most commonly with high-potency antipsychotic treatment and is best relieved by stopping the antipsychotic medication, providing med- ical support, and administering dantrolene, a muscle relaxant. After recovering from this life-threatening condition, the patient can be put on an atypical agent since they are less likely than high potency agents such as haloperidol to cause this dangerous side effect.

For the past few months, a 28-year-old woman has seemed full of energy and optimism for no obvious reason. Although she gets only about 6 hours of sleep a night, she has been very productive at work. She is talkative and gregarious and relates that she belongs to four clubs and two different sports teams. A few years previously, friends say she was often pessimistic and seemed tired and "washed out." During that period, she continued to work but did not seek out social activities and had little interest in sex. There is no evidence of a thought disorder and the patient denies suicidality or hopelessness. Physical examination including body weight is normal. 5. This patient shows evidence of (A) dysthymic disorder (B) major depressive disorder (C) bipolar disorder (D) hypochondriasis (E) cyclothymic disorder

The answer is E This patient shows evidence of cyclothymic disorder. This disorder involves periods of both hypomania (energy and optimism) and dysthymia (pessimism and feeling "washed out") occurring over a 2-year period with no discrete episodes of illness.

A 30-year-old patient is brought to the emergency department after being found running down the street naked. He is speaking very quickly and tells the physician that he has just given his clothing and all of his money to a homeless man. He states that God spoke to him and told him to do this. His history reveals that he is a practicing attorney who is married with three children. 16. The most effective immediate management for this patient is (A) lithium (B) fluoxetine (C) amitriptyline (D) diazepam (E) haloperidol

The answer is E This patient's good employment and relationship history suggest that his psychotic symptoms are an acute manifestation of a manic episode.

12. A 57-year-old male patient with a history of alcoholism has decided to stop drinking. Of the following, the agent most commonly used to treat anxiety and agitation associated with the initial stages of alcohol withdrawal is (A) zaleplon (B) flurazepam (C) clonazepam (D) buspirone (E) chlordiazepoxide (F) bupropion

The answer is E. Because it is long acting and has relatively low abuse potential for a BZ, chlordiazepoxide is the antianxiety agent most commonly used to treat the anxiety and agitation associated with the initial stages of alcohol withdrawal.

5. A 36-year-old man has been smoking two packs of cigarettes a day for the last 20 years. He is concerned that his health has deteriorated, and he has a persistent hacking cough. He also states that he doesn't want to "get lung cancer and die, like my father did." He has tried nicotine patches with no success and wants to know if there is any "pill" he could try. What medication could the physician recommend? (A) Mirtazapine (B) Citalopram (C) Phenelzine (D) Buspirone (E) Bupropion

The answer is E. Bupropion is an atypical heterocyclic antidepressant that is useful as an aid in smoking cessation. Mirtazapine is another atypical agent used in the treatment of depression that blocks both serotonin and α-adrenergic receptors. Phenelzine is an MAOI, sometimes used for depression. Citalopram is a selective serotonin reuptake inhibitor (SSRI), which are typically not used for smoking cessation. Buspirone is an antianxiety drug useful in situations where nonsedating agents are favored.

7. A patient has been using heroin for the past year. Which of the following is most likely to characterize this patient? (A) Age 48 years (B) Female gender (C) Insomnia when using the drug (D) Anxious mood when using the drug (E) Elevated mood when using the drug

The answer is E. Heroin users show an elevated, relaxed mood and somnolence. Users are most likely to be young adult males.

9. An anesthesia resident is on his first case alone. The surgeons are preparing the patient's abdomen for their eventual incision when the attending physician enters the operating room and asks the anesthesia resident if the patient is anesthetized. Which of the following is the most reliable sign that surgical anesthesia has been reached? (A) Analgesia (B) Amnesia (C) Loss of consciousness (D) Maximum papillary dilation (E) Loss of eyelash reflex

The answer is E. Loss of eyelash reflex and a pattern of respiration that is regular and deep are the most reliable indications of stage III, or surgical, anesthesia. Analgesia and amnesia are characteristics of stage I anesthesia, whereas the loss of consciousness is associated with stage II anesthesia. Maximum papillary dilation also occurs during stage III anesthesia, but closer to the progression to stage IV anesthesia, an undesirable stage associated with respiratory and cardiovascular failure.

A 29-year-old man comes to the emergency department complaining of stomach cramps, agitation, severe muscle aches, and diarrhea. Physical examination reveals that the patient is sweating, has dilated pupils, a fever, and a runny nose, and shows goose bumps on his skin. 2. Of the following, the most effective immediate treatment for relief of this patient's symptoms is (A) naloxone (B) naltrexone (C) anantipsychotic (D) a stimulant (E) clonidine

The answer is E. Of the choices given, the most effective immediate treatment for heroin withdrawal is *clonidine to stabilize the autonomic nervous system.* Psychotic symptoms are uncommon in opioid withdrawal and this patient does not need an antipsychotic. Naloxone and naltrexone as well as stimulants will worsen rather than ameliorate the patient's withdrawal symptoms.

Concerned parents of a 7-year-old boy bring their child to the pediatrician for evaluation. They note that ever since he was an infant, their son has never wanted to be held, cries whenever he is bathed, and becomes very upset when his daily routine is changed in any way. Although the child cannot yet read, his parents remark that he can identify the state of origin of any car license plate and almost exclusively plays with replicas of car license plates. The child's speaks in complete sen- tences and has a good vocabulary, but his behavior seems odd and he does not make eye contact when spoken to. Medical evaluation is unremarkable. 11. The major characteristic that suggests that this child has Asperger disorder rather than autistic disorder is that this child does not show (A) restricted interests (B) special abilities (C) focus on keeping up routines (D) problems in peer relationships (E) language delay

The answer is E. The major characteristic that differentiates autistic disorder from Asperger disorder is that in the former but not in the latter, there is developmental language delay. This child shows relatively normal language development, so Asperger disorder is more likely than autistic disorder in this case. Restricted or unusual interests (here, intense focus on state license plates), special abilities, focus on keeping up routines and problems in peer relationships are characteristic of both disorders.

A 62-year-old woman whose husband died 6 months ago tells her physician that she believes killing herself would end her suffering. Physical examination is unremarkable. 7. Of the following signs and symptoms, which is most likely to be seen in this patient? (A) Weight gain (B) Flight of ideas (C) Auditory hallucinations (D) Feeling better in the morning than in the evening (E) Poor grooming

The answer is E. This woman is showing evidence of major depression (note: Suicidality is not characteristic of a normal grief reaction). Depressed people typically show poor grooming. She is also more likely to show weight loss, and to feel better in the evening than in the morning. Auditory hallucinations are common in schizophrenia but uncommon in depression. Flight of ideas is characteristic of mania.

2. A 56-year-old truck driver is on disability for a back injury he sustained while making a delivery 3 months ago. He has been on several opioid drugs but continues to complain of "nagging back pain." The pain specialist he sees decides to try treating him with an antidepressant approved for the management of chronic pain. Which of the following would he chose? (A) Fluoxetine (B) Promethazine (C) Trazodone (D) Prochlorperazine (E) Venlafaxine

The answer is E. Venlafaxine and some tricyclic antidepressants are used in the management of chronic pain. Promethazine and prochlorperazine are two dopamine receptor blockers, without antipsychotic activity, used to treat nausea and vomiting. Trazodone is an atypical antidepressant that is highly sedating and has been associated with orthostatic hypoten- sion and priapism. Fluoxetine is a selective serotonin reuptake inhibitor (SSRI). SSRIs are not effective therapy for chronic pain.

7. A 3-year-old girl who had been develop- ing typically since birth begins to withdraw socially and then stops speaking altogether. Also, instead of purposeful hand movements, the child has begun to show repeti- tive hand wringing behavior. The chromosome most likely to be involved in this disorder is chromosome (A) 1 (B) 16 (C) 18 (D) 21 (E) X

The answer is E. This 3-year-old girl is showing signs of Rett's disorder which is linked to the X chromosome. Rett's disorder is characterized by loss of social skills after a period of typical functioning as well as hand wringing and breathing abnormalities (see also Chapter 15).

3. Which of the following drugs is (are), by self-report, the most frequently used in the United States? (A) Hallucinogens (B) Inhalants (C) Cocaine (D) Heroin (E) Marijuana

The answer is E. Almost 17 million Americans report that they use marijuana. In co trast, 1.6 million, 1.3 million, 0.6 million, and 0.2 million report that they use cocaine, hallucinogens, inhalants, and heroin, respectively.

20. Which of the following antidepressant agents is most likely to cause gynecomastia and parkinsonian symptoms in a 45-year-old male patient? (A) Venlafaxine (B) Tranylcypromine (C) Trazodone (D) Doxepin (E) Amoxapine (F) Fluoxetine (G) Nortriptyline (H) Imipramine

The answer is E. Amoxapine has antidopaminergic action and, thus, is the agent most likely to cause gynecomastia as well as parkinsonian symptoms in this patient.

10. A 53-year-old hospitalized patient with schizophrenia tells the physician that a newscaster was talking about her when he said on television, "A woman was found shoplifting today." This patient's statement is an example of (A) an illusion (B) a neologism (C) a hallucination (D) a delusion (E) an idea of reference

The answer is E. An idea of reference is the false belief of being referred to by others (e.g., a newscaster talking about the patient on television)

21. Analysis of the blood plasma of a 45-year-old male patient shows increased concentration of homovanillic acid (HVA). This elevation is most likely to be associated with which of the following conditions? (A) Parkinson disease (B) Depression (C) Bulimia (D) Pheochromocytoma (E) Schizophrenia

The answer is E. Increased body fluid level of homovanillic acid (HVA), a major metabolite of dopamine, is seen in schizophrenia. Decreased HVA is seen in Parkinson disease, depression, and in medicated schizophrenic patients. Increased vanillylmandelic acid (VMA), a metabolite of norepinephrine, is seen in pheochromocytoma. Decreased body fluid level of 5-HIAA, a metabolite of serotonin, is seen in depression and in bulimia (Table 4.5).

11. A 6-year-old child was badly burned when his house caught on fire. He sustained full-thickness burns on approximately 40% of his body. He has spent many months enduring multiple skin-grafting procedures. To aid in reducing the pain associated with dressing changes, he is given ketamine IV. This drug has been associated with which of the following adverse reactions? (A) Irritation to the respiratory airways (B) Sensitization of the heart to catecholamines (C) Reduction of cardiac output (D) Malignant hyperthermia (E) Distortion of reality and terrifying dreams

The answer is E. Ketamine is a dissociative anesthetic related to phencyclidine (PCP) and is thought to block glutamic acid N-methyl-d-aspartate (NMDA) receptors. Its use is associated with distortions of reality, terrifying dreams, and delirium, more commonly in adults. Malignant hyperthermia may be associated with any of the inhaled anesthetics, such as halothane, in genetically prone individuals. Halothane and isoflurane sensitize the heart to catecholamines. Desfluramine is especially irritating to airways, and enflurane can reduce cardiac output.

27. A resident is called to assess an oncology patient on a general medical floor who has developed a muscle spasm causing her neck to twist uncontrollably to the left. The intern evaluates the patient's list of medications and concludes that her new symptoms are probably due to which of the following? (A) Aspirin (B) Digoxin (C) Erythromycin (D) Fluoxetine (E) Metoclopramide

The answer is E. Metoclopramide (Reglan), a gastric motility agent and antiemetic, is often used to control nausea and vomiting in cancer patients receiving chemotherapy. It has antidopaminergic properties and can cause acute dystonic reactions such as are occurring in this patient. Management includes stopping the metoclopramide and providing an anticholinergic agent, such as benztropine, or an antihistamine, such as diphenhydramine, both of which are usually given in intramuscular form for immediate effect. Aspirin, digoxin, erythromycin, and fluoxetine are unlikely to cause dystonic reactions.

15. Which of the following is most closely associated with a good prognosis in schizo- phrenia? (A) Younger age of onset (B) Catatonic symptoms (C) Negative symptoms (D) Many relapses (E) Mood symptoms

The answer is E. Mood symptoms are associated with a good prognosis in schizophre- nia. A good prognosis is also associated with older age of onset, positive symptoms, and few relapses. Catatonic symptoms are associated with a poor prognosis.

7. When having a manic episode, a 53-year- old patient with bipolar disorder shows primary process thinking. This type of thinking (A) is logical (B) is closely attuned to time (C) is associated with reality (D) is accessible to the conscious mind (E) is associated with pleasure seeking

The answer is E. Primary process thinking is associated with pleasure seeking, disregards logic and reality, has no concept of time, and is not accessible to the conscious mind. Secondary process thinking is logical and is associated with reality.

8. Which of the following events is most likely to result in post-traumatic stress disorder (PTSD)? (A) Divorce (B) Bankruptcy (C) Diagnosis of diabetes mellitus (D) Changing residence (E) Robbery at knifepoint

The answer is E. Robbery at knifepoint, a life-threatening event, is most likely to result in post-traumatic stress disorder (PTSD). While life events such as divorce, bankruptcy, illness, and changing residence are stressful, they are rarely life-threatening. Psychological symptoms occurring after such less severe events may result in adjustment disorder, not PTSD.

8. When compared to traditional antipsy- chotic medication, atypical antipsychotic medication is more likely to be helpful for which of the following symptoms? (A) Hallucinations (B) Delusions (C) Agitation (D) Over-talkativeness (E) Social withdrawal

The answer is E. Social withdrawal is a negative symptom of schizophrenia. *Negative symptoms respond better to atypical antipsychotic medication than to traditional antipsychotics*. *Hallucinations, delusions, agitation, and over-talkativeness are positive symptoms of schizophrenia.*

25. A 45-year-old man presents in the emergency room with sinus tachycardia (112/ bpm), flattening of T waves and prolonged QT interval. The patient tells the physician that he is taking "nerve pills." Which of the following medications is this patient most likely to be taking? (A) Bupropion (B) Fluoxetine (C) Lorazepam (D) Valproic acid (E) Imipramine

The answer is E. TCAs such as imipramine cause sinus tachycardia, flat T waves, pro- longed QT interval and depressed ST segments. Bupropion, fluoxetine, lorazepam, and valproic acid are less likely to cause these cardiovascular effects.

1. A 30-year-old man who has had many negative life experiences becomes upset when he sees photographs of himself taken during these times. The brain area most likely to be activated by these photographs is the (A) dorsolateral convexity of the frontal lobe (B) hypothalamus (C) orbitofrontal cortex (D) reticular system (E) amygdala (F) nucleus basalis of Meynert

The answer is E. The amygdala is an important brain area for the evaluation of sensory stimuli with emotional significance. Thus, the brain area most likely to be activated by these photos is the amygdala.

15. A 30-year-old man with schizophrenia has been very withdrawn and apathetic for more than 10 years. He now is taking an antipsychotic agent that is helping him to be more outgoing and sociable. However, the patient is experiencing seizures, and agranulocytosis. The antipsychotic agent that this patient is most likely to be taking is (A) risperidone (B) thioridizine (C) olanzapine (D) haloperidol (E) clozapine

The answer is E. The antipsychotic agent that this patient is most likely to be taking is clozapine. Like other atypical agents, clozapine is more effective against negative symp- toms (e.g., withdrawal) than traditional agents such as haloperidol. However, clozapine is also is more likely to cause seizures and agranulocytosis than traditional agents or other atypicals, such as risperidone and olanzapine.

11. A 25-year-old male patient sustains a serious head injury in an automobile accident. He had been aggressive and assaultive, but after the accident he is placid and cooperative. He also makes inappropriate suggestive comments to the nurses and masturbates a great deal. The area(s) of the brain most likely to be affected in this patient is (are) the (A) right parietal lobe (B) basal ganglia (C) hippocampus (D) reticular system (E) amygdala (F) left frontal lobe

The answer is E. The patient is showing evidence of the Klüver-Bucy syndrome, which includes hypersexuality and docility and is associated with damage to the amygdala.

1. A 20-year-old man states that he is uncomfortable around women. He says that he gets anxious when he is with a woman and "just does not know what to say to her." The patient, a high school graduate, reports that he has a few male friends with whom he "hangs out" and is doing well in his job in construction. This clinical picture is most consistent with which of the following? (A) Schizoid personality disorder (B) Schizotypal personality disorder (C) Avoidant personality disorder (D) Asperger disorder (E) Normal shyness

The answer is E. This clinical picture is most consistent with normal shyness. Although this 20-year-old patient is somewhat anxious around women, the fact that he has friends and is doing well in his job makes it unlikely that he has a personality disorder or Asperger disorder

13. A 32-year-old man complains that he has no problem with erection, but that he usually has an orgasm and ejaculates before he achieves vaginal penetration. This man's complaint (A) is uncommon (B) is associated with depression (C) is associated with an absent excitement phase (D) can be effectively managed with intensive psychotherapy (E) can be effectively managed with the squeeze technique

The answer is E. This man is describing premature ejaculation, a common sexual dys- function, which can be effectively managed with the squeeze technique (not psycho- therapy). Premature ejaculation is associated with an absent plateau phase of the sexual response cycle and is not specifically associated with depression.

18. A 68-year-old patient tells the physician that for the last 7 years his neighbor has been trying to get him evicted from his apartment by telling lies about him to the landlord. The patient is married and is working full time in a job, which he has held for over 30 years. What is the most appropriate diagnosis for this patient? (A) Schizophrenia (B) Schizoaffective disorder (C) Schizophreniform disorder (D) Brief psychotic disorder (E) Delusional disorder (F) Shared psychoticdisorder (G) Psychosis due to a general medical condition

The answer is E. This patient is showing evidence of delusional disorder, persecutory type. In this disorder, there is a fixed, non-bizarre delusional system (paranoid in the persecutory type); few, if any, other thought disorders; and relatively normal social and occupational functioning (e.g., this patient is married and has held a job for over 30 years) (see also answer to Question 1).

12. The elderly parents of a 45-year-old mildly mentally retarded patient tell the physician that their son began to have difficulty identifying familiar objects and people about 6 months ago. Physical examination reveals that the patient is short in stature, has a protruding tongue, flat facies, hypotonia, and a thick neck. The chromosomal abnormality most likely to be responsible for this clinical picture is most likely to be associated with chromosome (A) 1 (B) 4 (C) 14 (D) 19 (E) 21

The answer is E. This patient with mild mental retardation and associated physical findings probably has Down syndrome, which is associated with chromosome 21. Down syndrome patients often develop Alzheimer disease in middle age, which explains the memory loss that this patient displays.

11. A 45-year-old woman has a 20-year history of vague physical complaints including nausea, painful menses, and loss of feeling in her legs. Physical examination and laboratory workup are unremarkable. She says that she has always had physical problems but her doctors never seem to identify their cause. (A) Post-traumatic stress disorder (B) Hypochondriasis (C) Obsessive-compulsive disorder (D) Panic disorder (E) Somatization disorder (F) Generalized anxiety disorder (G) Body dysmorphic disorder (H) Conversion disorder (I) Specific phobia (J) Social phobia (K) Adjustment disorder (L) Masked depression

The answer is E. This woman with a 20-year history of unexplained vague and chronic physical complaints probably has somatization disorder. This can be distinguished from hypochondriasis, which is an exaggerated worry about normal physical sensations and minor ailments

11. A 30-year-old single woman who has been smoking three packs of cigarettes a day for the last 10 years asks the physician to help her stop smoking. The doctor asks the patient why she smokes so much. The patient responds, "I always feel very alone and empty inside; I smoke to fill myself up." The patient reveals that she sometimes cuts the skin on her arms with a knife in order to "feel something." She also notes that when she is upset, she often uses cocaine and has sex with men whom she does not know well. After these episodes she typically feels even more alone and empty. Which of the following is the most characteristic defense mechanism used by people with this woman's personality characteristics? (A) Displacement (B) Intellectualization (C) Denial (D) Reaction formation (E) Splitting

The answer is E. This woman, who has always felt empty and alone (not merely lonely), shows evidence of borderline personality disorder. Borderline patients typically use splitting (see Chapter 6) as a defense mechanism. Self-injurious behavior and impulsive behavior (e.g., drug abuse, sex with multiple partners) also are characteristic of people with this personality disorder.

18. What is the most appropriate agent for a doctor to recommend for a 34-year-old, overweight, depressed patient who needs to take an antidepressant but is afraid of gaining weight? (A) Venlafaxine (B) Tranylcypromine (C) Trazodone (D) Doxepin (E) Amoxapine (F) Fluoxetine (G) Nortriptyline (H) Imipramine

The answer is F. *In contrast to most antidepressant agents, which are associated with weight gain, fluoxetine (Prozac) is associated with some weight loss.* Thus it is the most appropriate antidepressant agent for a patient who is afraid of gaining weight.

25. A 40-year-old patient with bruises on his arms, neck, and back tells the doctor that his lover often berates him and physically abuses him. He begs the doctor not to interfere because he is afraid that the man will desert him and that he will be alone. (A) Borderline personality disorder (B) Histrionic personality disorder (C) Obsessive-compulsive personality disorder (D) Avoidant personality disorder (E) Antisocial personality disorder (F) Dependent personality disorder (G) Dissociative identity disorder (H) Paranoid personality disorder (I) Passive-aggressive personality disorder (J) Narcissistic personality disorder (K) Schizotypal personality disorder (L) Schizoid personality disorder

The answer is F. This abused man is showing evidence of dependent personality disor- der. He tolerates his partner's abuse because of his overriding fear of being deserted by his lover, being alone, and having to make his own decisions.

16. A 45-year-old male patient becomes depressed following a head injury. The area(s) of the brain most likely to be affected in this patient is (are) the (A) right parietal lobe (B) basal ganglia (C) hippocampus (D) reticular system (E) amygdala (F) left frontal lobe

The answer is F. Of the listed brain areas, depression is most likely to be associated with damage to the left frontal lobe.

23. A man and woman in their mid 20s who have been married for 3 years come to a physician for evaluation of infertility. During the interview the wife states, "I cannot understand why I cannot get pregnant. We have had sexual relations two to three times per week for the past year." What is the most appropriate next step for the physician to take? (A) Refer the couple for marital counseling (B) Perform a gynecological exam on the wife (C) Perform a testicular exam on the husband (D) Suggest that the husband undergo a fertility evaluation (E) Suggest that the wife undergo a fertility evaluation (F) Ask the couple what they mean by "sexual relations"

The answer is F. The most appropriate next step for the physician to take is to clarify what the couple means by "sexual relations." Sexually inexperienced people may not know that some forms of sexual expression (e.g., fellatio, intercourse without ejaculation) cannot result in pregnancy. It is inappropriate to conduct physical or laboratory examinations for the cause of infertility until the physician is assured that the couple is having sexual intercourse involving vaginal penetration and ejaculation.

19. A 35-year-old nurse is brought to the emergency room after fainting outside of a patient's room. The nurse notes that she has had fainting episodes before and that she often feels weak and shaky. Laboratory studies reveal hypoglycemia, very high insulin level, and suppressed plasma C peptide. Which of the following best fits this clinical picture? (A) A sleep disorder (B) An anxiety (C) A somatoform disorder (D) Malingering (E) An endocrine disorder (F) A factitious disorder

The answer is F. The triad of hypoglycemia, very high insulin level, and suppressed plasma C peptide indicates that this nurse has self-administered insulin, a situation known as factitious hyperinsulinism. In hyperinsulinism due to medical causes, for example, insulinoma (pancreatic B-cell tumor), plasma C peptide is typically increased, not decreased. Factitious disorder is more common in people associated with the health professions. There is no evidence in this woman of a sleep disorder, anxiety dis- order, somatoform disorder, or endocrine disorder such as diabetes. Because there is no obvious or practical gain for this woman in being ill, malingering is unlikely.

19. A 60-year-old woman whose husband believes (in the absence of any evidence) that their house is filled with radioactive dust worries about her ability to clear the house of the dust when he is hospitalized. What is the most appropriate diagnosis for this woman? (A) Schizophrenia (B) Schizoaffective disorder (C) Schizophreniform disorder (D) Brief psychotic disorder (E) Delusional disorder (F) Shared psychotic disorder (G) Psychosis due to a general medical condition

The answer is F. This patient is showing evidence of shared psychotic disorder. She has developed the same delusion that her husband has (i.e., their house is filled with radioactive dust). If separated for a period of time from her husband (the inducer), her psychotic symptoms are likely to remit (see also answer to Question 1).

7. A 39-year-old woman claims that she injured her hand at work. She asserts that the pain caused by her injury prevents her from working. She has no further hand problems after she receives a $30,000 workers' compensation settlement. This clinical presentation is an example of (A) factitious disorder (B) conversion disorder (C) factitious disorder by proxy (D) somatization disorder (E) somatoform pain disorder (F) malingering

The answer is F. This presentation is an example of malingering, feigning illness for obvious gain (the $30,000 workers' compensation settlement). Evidence for this is that the woman has no further hand problems after she receives the money. In conversion disorder, somatization disorder, factitious disorder, and factitious disorder by proxy there is no obvious or material gain related to the symptoms.

8. The major neurotransmitter implicated in both Alzheimer disease and schizophrenia is (A) serotonin (B) norepinephrine (C) dopamine (D) g-aminobutyric acid (GABA) (E) acetylcholine (Ach) (F) glutamate

The answer is F. While acetylcholine (Ach) is the major neurotransmitter implicated in Alzheimer disease, abnormalities in glutamate are seen in both Alzheimer disease and schizophrenia.

16. A 28-year-old woman seeks facial reconstructive surgery for her "sagging" eyelids. She rarely goes out in the daytime because she believes that this characteristic makes her look "like a grandmother." On physical examination, her eyelids appear completely normal. (A) Post-traumatic stress disorder (B) Hypochondriasis (C) Obsessive-compulsive disorder (D) Panic disorder (E) Somatization disorder (F) Generalized anxiety disorder (G) Body dysmorphic disorder (H) Conversion disorder (I) Specific phobia (J) Social phobia (K) Adjustment disorder (L) Masked depression

The answer is G. This woman probably has body dysmorphic disorder, which is characterized by over-concern about a physical feature (e.g., "sagging" eyelids in this case), despite normal appearance.

19. A 32-year-old man is brought to a New York City hospital. He appears sedated, but shows an elevated mood. A blood test reveals the presence of HIV. (A) Alcohol (B) Secobarbital (C) Cocaine (D) Methylphenidate (E) Caffeine (F) Diazepam (G) Heroin (H) Marijuana (I) Nicotine (J) Phencyclidine (PCP) (K) Lysergic acid diethylamide (LSD)

The answer is G. The presence of HIV as well as signs of sedation and euphoria indicate that this patient is an intravenous heroin user.

18. The tenting effect is most likely to begin in which stage of the sexual response cycle and in men only, women only, or both men and women? (A) Excitement phase: Men only (B) Plateau phase: Men only (C) Orgasm phase:Men only (D) Excitement phase: Men and women (E) Plateau phase: Men and women (F) Orgasm phase: Men and women (G) Excitement phase: Women only (H) Plateau phase: Women only (I) Orgasm phase: Women only

The answer is G. The tenting effect, elevation of the uterus in the pelvic cavity, begins during the excitement phase of the sexual response cycle in women.

13. A 29-year-old man experiences sudden right-sided hemiparesis, but appears unconcerned. He reports that just before the onset of weakness, he saw his girlfriend with another man. Physical examination fails to reveal evidence of a medical problem. (A) Post-traumatic stress disorder (B) Hypochondriasis (C) Obsessive-compulsive disorder (D) Panic disorder (E) Somatization disorder (F) Generalized anxiety disorder (G) Body dysmorphic disorder (H) Conversion disorder (I) Specific phobia (J) Social phobia (K) Adjustment disorder (L) Masked depression

The answer is H. This man, who experiences a sudden neurological symptom triggered by seeing his girlfriend with another man, is showing evidence of conversion disorder. *This disorder is characterized by an apparent lack of concern about the symptoms (i.e., la belle indifference).*

22. Two weeks after a 50-year-old, over- weight, hypertensive woman agreed to start an exercise program, she gained 4 pounds. She reports that she has not exercised yet because "the gym was too crowded." (A) Borderline personality disorder (B) Histrionic personality disorder (C) Obsessive-compulsive personality disorder (D) Avoidant personality disorder (E) Antisocial personality disorder (F) Dependent personality disorder (G) Dissociative identity disorder (H) Paranoid personality disorder (I) Passive-aggressive personality disorder (J) Narcissistic personality disorder (K) Schizotypal personality disorder (L) Schizoid personality disorder

The answer is I. This 50-year-old woman, who agreed to start an exercise program and then makes weak excuses for her failure to follow the program, is showing evidence of passive-aggressive personality disorder. She did not really want to follow the doctor's exercise program (was inwardly defiant) but agreed to do it (was outwardly compliant).

22. A 28-year-old man is hospitalized after trying to jump from the roof of one apart- ment building to another. His friends relate that prior to the jump, the man angrily threatened them because they would not jump with him. (A) Alcohol (B) Secobarbital (C) Cocaine (D) Methylphenidate (E) Caffeine (F) Diazepam (G) Heroin (H) Marijuana (I) Nicotine (J) Phencyclidine (PCP) (K) Lysergic acid diethylamide (LSD)

The answer is J. *Aggressiveness and psychotic behavior* (jumping from one rooftop to another) indicate that this patient has used PCP.

19. A 38-year-old man asks his doctor to refer him to a physician who attended a top-rated medical school. He says that he knows the doctor will not be offended because she will understand that he is "better" than her other patients. (A) Borderline personality disorder (B) Histrionic personality disorder (C) Obsessive-compulsive personality disorder (D) Avoidant personality disorder (E) Antisocial personality disorder (F) Dependent personality disorder (G) Dissociative identity disorder (H) Paranoid personality disorder (I) Passive-aggressive personality disorder (J) Narcissistic personality disorder (K) Schizotypal personality disorder (L) Schizoid personality disorder

The answer is J. This 38-year-old man, who asks to be referred to a physician who attended a top-rated medical school because he is "better" than other patients, is showing evidence of narcissistic personality disorder

17. A 29-year-old man is upset because he must take a client to dinner in a restaurant. Although he knows the client well, he is so afraid of making a mess while eating that he says he is not hungry and sips from a glass of water instead of ordering a meal. (A) Post-traumatic stress disorder (B) Hypochondriasis (C) Obsessive-compulsive disorder (D) Panic disorder (E) Somatization disorder (F) Generalized anxiety disorder (G) Body dysmorphic disorder (H) Conversion disorder (I) Specific phobia (J) Social phobia (K) Adjustment disorder (L) Masked depression

The answer is J. This man probably has social phobia. He is afraid of embarrassing himself in a public situation (e.g., getting food on his face while eating dinner in front of others in a restaurant).

12. Three months after moving, a teenager who was formerly outgoing and a good student seems sad, loses interest in making friends, and begins to do poor work in school. His appetite is normal and there is no evidence of suicidal ideation. (A) Post-traumatic stress disorder (B) Hypochondriasis (C) Obsessive-compulsive disorder (D) Panic disorder (E) Somatization disorder (F) Generalized anxiety disorder (G) Body dysmorphic disorder (H) Conversion disorder (I) Specific phobia (J) Social phobia (K) Adjustment disorder (L) Masked depression

The answer is K. This teenager, who was formerly outgoing and a good student and now seems sad, loses interest in making friends, and begins to do poor work in school, probably has adjustment disorder (with depressed mood). It is likely that he is having problems adjusting to his new school. In contrast to adjustment disorder, in masked depression the symptoms are more severe and often include significant weight loss or suicidality

18. A 29-year-old man tells the doctor that he has been so "nervous" and upset since his girlfriend broke up with him 1 month ago that he had to quit his job and stay at home. The man has no history of medical or psychiatric disorders, although his father has a history of bipolar disorder, his mother has a history of alcoholism, and his younger brother was in rehab for drug abuse the previous year. (A) Post-traumatic stress disorder (B) Hypochondriasis (C) Obsessive-compulsive disorder (D) Panic disorder (E) Somatization disorder (F) Generalized anxiety disorder (G) Body dysmorphic disorder (H) Conversion disorder (I) Specific phobia (J) Social phobia (K) Adjustment disorder (L) Masked depression

The answer is K. The most likely explanation for this clinical picture that includes symptoms of anxiety which begin after a life stressor (e.g., a romantic break-up) is adjustment disorder (with anxiety). The absence of a previous history and the brief duration indicates that this is not an anxiety disorder and the fact that the stressor was not life-threatening rules out PTSD and ASD. The family history is not likely to be related to this patient's symptoms in this case.

23. A 22-year-old woman is brought to the emergency room at 8 AM by her friend, who states that the woman has been acting strangely since the previous evening. While lying on the examining table the patient states that she feels like she is floating in the air and the sun is big and glaring above her. (A) Alcohol (B) Secobarbital (C) Cocaine (D) Methylphenidate (E) Caffeine (F) Diazepam (G) Heroin (H) Marijuana (I) Nicotine (J) Phencyclidine (PCP) (K) Lysergic acid diethylamide (LSD)

The answer is K. This woman, who has been acting strangely over a number of hours and is *experiencing out-of-body experiences* (e.g., feelings of floating in the air) and illusions (e.g., mistaking the overhead light for the sun), has probably taken LSD. The patient's lack of aggression or agitation indicates that the hallucinogen she has used is less likely to have been PCP.

23. The parents of a 26-year-old woman say that they are concerned about her because she has no friends and spends most of her time hiking in the woods and working on her computer. The doctor examines her and finds that she is content with her solitary life and has no evidence of a thought disorder. (A) Borderline personality disorder (B) Histrionic personality disorder (C) Obsessive-compulsive personality disorder (D) Avoidant personality disorder (E) Antisocial personality disorder (F) Dependent personality disorder (G) Dissociative identity disorder (H) Paranoid personality disorder (I) Passive-aggressive personality disorder (J) Narcissistic personality disorder (K) Schizotypal personality disorder (L) Schizoid personality disorder

The answer is L. This 26-year-old woman, who shows no evidence of a thought disorder, has no friends, and spends most of her time at solitary pursuits, is showing evidence of schizoid personality disorder. Patients with schizoid personality disorder are typically content with their solitary lifestyle.

15. A 41-year-old man says that he has been "sickly" for the past 3 months. He fears that he has stomach cancer. The patient is unshaven and appears thin and slowed down. Physical examination, including a gastrointestinal workup, is unremarkable except that there is an unexplained loss of 15 pounds since his last visit 1 year ago. (A) Post-traumatic stress disorder (B) Hypochondriasis (C) Obsessive-compulsive disorder (D) Panic disorder (E) Somatization disorder (F) Generalized anxiety disorder (G) Body dysmorphic disorder (H) Conversion disorder (I) Specific phobia (J) Social phobia (K) Adjustment disorder (L) Masked depression

The answer is L. This man probably has masked depression. In contrast to the hypochondriacal man in the previous question, evidence for depression in this patient includes the fact that, in addition to the somatic complaints, he shows symptoms of depression (e.g., he is not groomed, appears slowed down [psychomotor retardation], and has lost a significant amount of weight).


Set pelajaran terkait

Chapter 33: Disorders of Renal Function - ML3

View Set

Chpt 17-Mental Health Care for Survivor of Violence

View Set

MED SURG SUCCESS (RESPIRATORY Q's)

View Set

University of Idaho: Microeconomics 202 Ch. 13

View Set

Capitals of Spanish Speaking Countries

View Set